What are your chances of acceptance?

Calculate for all schools, your chance of acceptance.

Duke University

Your chancing factors

Extracurriculars.

scoring rubric for synthesis essay

How to Write the AP Lang Synthesis Essay + Example

Do you know how to improve your profile for college applications.

See how your profile ranks among thousands of other students using CollegeVine. Calculate your chances at your dream schools and learn what areas you need to improve right now — it only takes 3 minutes and it's 100% free.

Show me what areas I need to improve

What’s Covered:

What is the ap lang synthesis essay, how will ap scores affect my college chances.

AP English Language and Composition, commonly known as AP Lang, is one of the most engaging and popular AP classes offered at most high schools, with over 535,000 students taking the class . AP Lang tests your ability to analyze written pieces, synthesize information, write rhetorical essays, and create cohesive and concrete arguments. However, the class is rather challenging as only 62% of students were able to score a three or higher on the exam. 

The AP Lang exam has two sections. The first consists of 45 multiple choice questions which need to be completed in an hour. This portion counts for around 45% of your total score. These questions ask students to analyze written pieces and answer questions related to each respective passage.  All possible answer choices can be found within the text, and no prior knowledge of literature is needed to understand the passages.

The second section contains three free-response questions to be finished in under two hours and 15 minutes. This section counts for 55% of your score and includes the synthesis essay, the rhetorical essay, and the argumentative essay.

  • The synthesis essay requires you to read 6-7 sources and create an argument using at least three sources.
  • The rhetorical analysis essay requires you to describe how a piece of writing evokes specific meanings and symbolism.
  • The argumentative essay requires you to pick a perspective of a debate and create an argument based on the evidence provided.

In this post, we will take a look at the AP Lang synthesis essay and discuss tips and tricks to master this part of the exam. We will also provide an example of a well-written essay for review.  

The AP Lang synthesis essay is the first of three essays included in the Free Response section of the AP Lang exam. The exam presents 6-7 sources that are organized around a specific topic, with two of those sources purely visual, including a single quantitative source (like a graph or pie chart). The remaining 4-5 sources are text-based, containing around 500 words each. It’s recommended that students spend an hour on this essay—15 minute reading period, 40 minutes writing, and 5 minutes of spare time to check over work.

Each synthesis essay has a topic that all the sources will relate to. A prompt will explaining the topic and provide some background, although the topics are usually broad so you will probably know something related to the issue. It will also present a claim that students will respond to in an essay format using information from at least three of the provided sources. You will need to take a stance, either agreeing or disagreeing with the position provided in the claim. 

According to the CollegeBoard, they are looking for essays that “combine different perspectives from sources to form a support of a coherent position.” This means that you must state your claim on the topic and highlight relationships between several sources that support your specific position on the topic. Additionally, you’ll need to cite clear evidence from your sources to prove your point.

The synthesis essay counts for six points on the AP Lang exam. Students can receive 0-1 points for writing a thesis statement, 0-4 based on the incorporation of evidence and commentary, and 0-1 points based on the sophistication of thought and demonstration of complex understanding.

While this essay seems extremely overwhelming, considering there are a total of three free-response essays to complete, with proper time management and practiced skills, this essay is manageable and straightforward. In order to enhance the time management aspect of the test to the best of your ability, it is essential to divide the essay up into five key steps.

Step 1: Analyze the Prompt

As soon as the clock starts, carefully read and analyze what the prompt asks from you. It might be helpful to markup the text to identify the most critical details. You should only spend around 2 minutes reading the prompt so you have enough time to read all the sources and figure out your argument. Don’t feel like you need to immediately pick your stance on the claim right after reading the prompt. You should read the sources before you commit to your argument.

Step 2: Read the Sources Carefully

Although you are only required to use 3 of the 6-7 sources provides, make sure you read ALL of the sources. This will allow you to better understand the topic and make the most educated decision of which sources to use in your essay. Since there are a lot of sources to get through, you will need to read quickly and carefully.

Annotating will be your best friend during the reading period. Highlight and mark important concepts or lines from each passage that would be helpful in your essay. Your argument will probably begin forming in your head as you go through the passages, so you will save yourself a lot of time later on if you take a few seconds to write down notes in the margins. After you’ve finished reading a source, reflect on whether the source defends, challenges, or qualifies your argument.

You will have around 13 minutes to read through all the sources, but it’s very possible you will finish earlier if you are a fast reader. Take the leftover time to start developing your thesis and organizing your thoughts into an outline so you have more time to write. 

Step 3: Write a Strong Thesis Statement 

In order to write a good thesis statement, all you have to do is decide your stance on the claim provided in the prompt and give an overview of your evidence. You essentially have three choices on how to frame your thesis statement: You can defend, challenge or qualify a claim that’s been provided by the prompt. 

  • If you are defending the claim, your job will be to prove that the claim is correct .
  • If you are challenging the claim, your job will be to prove that the claim is incorrect .
  • If you choose to qualify the claim, your job will be to agree to a part of the claim and disagree with another part of the claim. 

A strong thesis statement will clearly state your stance without summarizing the issue or regurgitating the claim. The CollegeBoard is looking for a thesis statement that “states a defensible position and establishes a line of reasoning on the issue provided in the prompt.”

Step 4: Create a Minimal Essay Outline

Developing an outline might seem like a waste of time when you are up against the clock, but believe us, taking 5-10 minutes to outline your essay will be much more useful in the long run than jumping right into the essay.

Your outline should include your thesis statement and three main pieces of evidence that will constitute each body paragraph. Under each piece of evidence should be 2-3 details from the sources that you will use to back up your claim and some commentary on how that evidence proves your thesis.

Step 5: Write your Essay

Use the remaining 30-35 minutes to write your essay. This should be relatively easy if you took the time to mark up the sources and have a detailed outline.  Remember to add special consideration and emphasis to the commentary sections of the supporting arguments outlined in your thesis. These sentences are critical to the overall flow of the essay and where you will be explaining how the evidence supports or undermines the claim in the prompt.

Also, when referencing your sources, write the in-text citations as follows: “Source 1,” “Source 2,” “Source 3,” etc. Make sure to pay attention to which source is which in order to not incorrectly cite your sources. In-text citations will impact your score on the essay and are an integral part of the process.

After you finish writing, read through your essay for any grammatical errors or mistakes before you move onto the next essay.

Here are six must-have tips and tricks to get a good score on the synthesis essay:

  • Cite at least four sources , even though the minimum requirement is three. Remember not to plagiarize and cite everything you use in your arguments.
  • Make sure to develop a solid and clear thesis . Develop a stable stance for the claim and stick with it throughout the entire paper.
  • Don’t summarize the sources. The summary of the sources does not count as an argument. 
  • You don’t necessarily have to agree with the sources in order to cite them. Using a source to support a counterargument is still a good use of a source.
  • Cite the sources that you understand entirely . If you don’t, it could come back to bite you in the end. 
  • Use small quotes , do not quote entire paragraphs. Make sure the quote does not disrupt the flow or grammar of the sentence you write. 

scoring rubric for synthesis essay

Discover your chances at hundreds of schools

Our free chancing engine takes into account your history, background, test scores, and extracurricular activities to show you your real chances of admission—and how to improve them.

Here is an example prompt and essay from 2019 that received 5 of the 6 total points available:

In response to our society’s increasing demand for energy, large-scale wind power has drawn attention from governments and consumers as a potential alternative to traditional materials that fuel our power grids, such as coal, oil, natural gas, water, or even newer sources such as nuclear or solar power. Yet the establishment of large-scale, commercial-grade wind farms is often the subject of controversy for a variety of reasons.

Carefully read the six sources, found on the AP English Language and Composition 2019 Exam (Question 1), including the introductory information for each source. Write an essay that synthesizes material from at least three of the sources and develops your position on the most important factors that an individual or agency should consider when deciding whether to establish a wind farm.

Source A (photo)

Source B (Layton)

Source C (Seltenrich)

Source D (Brown)

Source E (Rule)

Source F (Molla)

In your response you should do the following:

  • Respond to the prompt with a thesis presents a defensible position.
  • Select and use evidence from at least 3 of the provided sources to support your line of reasoning. Indicate clearly the sources used through direct quotation, paraphrase, or summary. Sources may be cited as Source A, Source B, etc., or by using the description in parentheses.
  • Explain how the evidence supports your line of reasoning.
  • Use appropriate grammar and punctuation in communicating your argument.

[1] The situation has been known for years, and still very little is being done: alternative power is the only way to reliably power the changing world. The draw of power coming from industry and private life is overwhelming current sources of non-renewable power, and with dwindling supplies of fossil fuels, it is merely a matter of time before coal and gas fuel plants are no longer in operation. So one viable alternative is wind power. But as with all things, there are pros and cons. The main factors for power companies to consider when building wind farms are environmental boon, aesthetic, and economic factors.

[2] The environmental benefits of using wind power are well-known and proven. Wind power is, as qualified by Source B, undeniably clean and renewable. From their production requiring very little in the way of dangerous materials to their lack of fuel, besides that which occurs naturally, wind power is by far one of the least environmentally impactful sources of power available. In addition, wind power by way of gearbox and advanced blade materials, has the highest percentage of energy retention. According to Source F, wind power retains 1,164% of the energy put into the system – meaning that it increases the energy converted from fuel (wind) to electricity 10 times! No other method of electricity production is even half that efficient. The efficiency and clean nature of wind power are important to consider, especially because they contribute back to power companies economically.

[3] Economically, wind power is both a boon and a bone to electric companies and other users. For consumers, wind power is very cheap, leading to lower bills than from any other source. Consumers also get an indirect reimbursement by way of taxes (Source D). In one Texan town, McCamey, tax revenue increased 30% from a wind farm being erected in the town. This helps to finance improvements to the town. But, there is no doubt that wind power is also hurting the power companies. Although, as renewable power goes, wind is incredibly cheap, it is still significantly more expensive than fossil fuels. So, while it is helping to cut down on emissions, it costs electric companies more than traditional fossil fuel plants. While the general economic trend is positive, there are some setbacks which must be overcome before wind power can take over as truly more effective than fossil fuels.

[4] Aesthetics may be the greatest setback for power companies. Although there may be significant economic and environmental benefit to wind power, people will always fight to preserve pure, unspoiled land. Unfortunately, not much can be done to improve the visual aesthetics of the turbines. White paint is the most common choice because it “[is] associated with cleanliness.” (Source E). But, this can make it stand out like a sore thumb, and make the gargantuan machines seem more out of place. The site can also not be altered because it affects generating capacity. Sound is almost worse of a concern because it interrupts personal productivity by interrupting people’s sleep patterns. One thing for power companies to consider is working with turbine manufacturing to make the machines less aesthetically impactful, so as to garner greater public support.

[5] As with most things, wind power has no easy answer. It is the responsibility of the companies building them to weigh the benefits and the consequences. But, by balancing economics, efficiency, and aesthetics, power companies can create a solution which balances human impact with environmental preservation.

More examples can be found here at College Board.

While AP Scores help to boost your weighted GPA, or give you the option to get college credit, AP Scores don’t have a strong effect on your admissions chances . However, colleges can still see your self-reported scores, so you might not want to automatically send scores to colleges if they are lower than a 3. That being said, admissions officers care far more about your grade in an AP class than your score on the exam.

Related CollegeVine Blog Posts

scoring rubric for synthesis essay

PrepScholar

Choose Your Test

Sat / act prep online guides and tips, how to write a perfect synthesis essay for the ap language exam.

author image

Advanced Placement (AP)

body-pencil-sharpen-notebook-1

If you're planning to take the AP Language (or AP Lang) exam , you might already know that 55% of your overall exam score will be based on three essays. The first of the three essays you'll have to write on the AP Language exam is called the "synthesis essay." If you want to earn full points on this portion of the AP Lang Exam, you need to know what a synthesis essay is and what skills are assessed by the AP Lang synthesis essay.

In this article, we'll explain the different aspects of the AP Lang synthesis essay, including what skills you need to demonstrate in your synthesis essay response in order to achieve a good score. We'll also give you a full breakdown of a real AP Lang Synthesis Essay prompt, provide an analysis of an AP Lang synthesis essay example, and give you four tips for how to write a synthesis essay.

Let's get started by taking a closer look at how the AP Lang synthesis essay works!

Synthesis Essay AP Lang: What It Is and How It Works

The AP Lang synthesis essay is the first of three essays included in the Free Response section of the AP Lang exam.

The AP Lang synthesis essay portion of the Free Response section lasts for one hour total . This hour consists of a recommended 15 minute reading period and a 40 minute writing period. Keep in mind that these time allotments are merely recommendations, and that exam takers can parse out the allotted 60 minutes to complete the synthesis essay however they choose.

Now, here's what the structure of the AP Lang synthesis essay looks like. The exam presents six to seven sources that are organized around a specific topic (like alternative energy or eminent domain, which are both past synthesis exam topics).

Of these six to seven sources, at least two are visual , including at least one quantitative source (like a graph or pie chart, for example). The remaining four to five sources are print text-based, and each one contains approximately 500 words.

In addition to six to seven sources, the AP Lang exam provides a written prompt that consists of three paragraphs. The prompt will briefly explain the essay topic, then present a claim that students will respond to in an essay that synthesizes material from at least three of the sources provided.

Here's an example prompt provided by the College Board:

Directions : The following prompt is based on the accompanying six sources.

This question requires you to integrate a variety of sources into a coherent, well-written essay. Refer to the sources to support your position; avoid mere paraphrase or summary. Your argument should be central; the sources should support this argument .

Remember to attribute both direct and indirect citations.

Introduction

Television has been influential in United States presidential elections since the 1960's. But just what is this influence, and how has it affected who is elected? Has it made elections fairer and more accessible, or has it moved candidates from pursuing issues to pursuing image?

Read the following sources (including any introductory information) carefully. Then, in an essay that synthesizes at least three of the sources for support, take a position that defends, challenges, or qualifies the claim that television has had a positive impact on presidential elections.

Refer to the sources as Source A, Source B, etc.; titles are included for your convenience.

Source A (Campbell) Source B (Hart and Triece) Source C (Menand) Source D (Chart) Source E (Ranney) Source F (Koppel)

Like we mentioned earlier, this prompt gives you a topic — which it briefly explains — then asks you to take a position. In this case, you'll have to choose a stance on whether television has positively or negatively affected U.S. elections. You're also given six sources to evaluate and use in your response. Now that you have everything you need, now your job is to write an amazing synthesis essay.

But what does "synthesize" mean, exactly? According to the CollegeBoard, when an essay prompt asks you to synthesize, it means that you should "combine different perspectives from sources to form a support of a coherent position" in writing. In other words, a synthesis essay asks you to state your claim on a topic, then highlight the relationships between several sources that support your claim on that topic. Additionally, you'll need to cite specific evidence from your sources to prove your point.

The synthesis essay counts for six of the total points on the AP Lang exam . Students can receive 0-1 points for writing a thesis statement in the essay, 0-4 based on incorporation of evidence and commentary, and 0-1 points based on sophistication of thought and demonstrated complex understanding of the topic.

You'll be evaluated based on how effectively you do the following in your AP Lang synthesis essay:

Write a thesis that responds to the exam prompt with a defensible position

Provide specific evidence that to support all claims in your line of reasoning from at least three of the sources provided, and clearly and consistently explain how the evidence you include supports your line of reasoning

Demonstrate sophistication of thought by either crafting a thoughtful argument, situating the argument in a broader context, explaining the limitations of an argument

Make rhetorical choices that strengthen your argument and/or employ a vivid and persuasive style throughout your essay.

If your synthesis essay meets the criteria above, then there's a good chance you'll score well on this portion of the AP Lang exam!

If you're looking for even more information on scoring, the College Board has posted the AP Lang Free Response grading rubric on its website. ( You can find it here. ) We recommend taking a close look at it since it includes additional details about the synthesis essay scoring.

body-chisel-break-apart

Don't be intimidated...we're going to teach you how to break down even the hardest AP synthesis essay prompt.

Full Breakdown of a Real AP Lang Synthesis Essay Prompt

In this section, we'll teach you how to analyze and respond to a synthesis essay prompt in five easy steps, including suggested time frames for each step of the process.

Step 1: Analyze the Prompt

The very first thing to do when the clock starts running is read and analyze the prompt. To demonstrate how to do this, we'll look at the sample AP Lang synthesis essay prompt below. This prompt comes straight from the 2018 AP Lang exam:

Eminent domain is the power governments have to acquire property from private owners for public use. The rationale behind eminent domain is that governments have greater legal authority over lands within their dominion than do private owners. Eminent domain has been instituted in one way or another throughout the world for hundreds of years.

Carefully read the following six sources, including the introductory information for each source. Then synthesize material from at least three of the sources and incorporate it into a coherent, well-developed essay that defends, challenges, or qualifies the notion that eminent domain is productive and beneficial.

Your argument should be the focus of your essay. Use the sources to develop your argument and explain the reasoning for it. Avoid merely summarizing the sources. Indicate clearly which sources you are drawing from, whether through direct quotation, paraphrase, or summary. You may cite the sources as Source A, Source B, etc., or by using the descriptions in parentheses.

On first read, you might be nervous about how to answer this prompt...especially if you don't know what eminent domain is! But if you break the prompt down into chunks, you'll be able to figure out what the prompt is asking you to do in no time flat.

To get a full understanding of what this prompt wants you to do, you need to identify the most important details in this prompt, paragraph by paragraph. Here's what each paragraph is asking you to do:

  • Paragraph 1: The prompt presents and briefly explains the topic that you'll be writing your synthesis essay about. That topic is the concept of eminent domain.
  • Paragraph 2: The prompt presents a specific claim about the concept of eminent domain in this paragraph: Eminent domain is productive and beneficial. This paragraph instructs you to decide whether you want to defend, challenge, or qualify that claim in your synthesis essay , and use material from at least three of the sources provided in order to do so.
  • Paragraph 3: In the last paragraph of the prompt, the exam gives you clear instructions about how to approach writing your synthesis essay . First, make your argument the focus of the essay. Second, use material from at least three of the sources to develop and explain your argument. Third, provide commentary on the material you include, and provide proper citations when you incorporate quotations, paraphrases, or summaries from the sources provided.

So basically, you'll have to agree with, disagree with, or qualify the claim stated in the prompt, then use at least three sources substantiate your answer. Since you probably don't know much about eminent domain, you'll probably decide on your position after you read the provided sources.

To make good use of your time on the exam, you should spend around 2 minutes reading the prompt and making note of what it's asking you to do. That will leave you plenty of time to read the sources provided, which is the next step to writing a synthesis essay.

Step 2: Read the Sources Carefully

After you closely read the prompt and make note of the most important details, you need to read all of the sources provided. It's tempting to skip one or two sources to save time--but we recommend you don't do this. That's because you'll need a thorough understanding of the topic before you can accurately address the prompt!

For the sample exam prompt included above, there are six sources provided. We're not going to include all of the sources in this article, but you can view the six sources from this question on the 2018 AP Lang exam here . The sources include five print-text sources and one visual source, which is a cartoon.

As you read the sources, it's important to read quickly and carefully. Don't rush! Keep your pencil in hand to quickly mark important passages that you might want to use as evidence in your synthesis. While you're reading the sources and marking passages, you want to think about how the information you're reading influences your stance on the issue (in this case, eminent domain).

When you finish reading, take a few seconds to summarize, in a phrase or sentence, whether the source defends, challenges, or qualifies whether eminent domain is beneficial (which is the claim in the prompt) . Though it might not feel like you have time for this, it's important to give yourself these notes about each source so you know how you can use each one as evidence in your essay.

Here's what we mean: say you want to challenge the idea that eminent domain is useful. If you've jotted down notes about each source and what it's saying, it will be easier for you to pull the relevant information into your outline and your essay.

So how much time should you spend reading the provided sources? The AP Lang exam recommends taking 15 minutes to read the sources . If you spend around two of those minutes reading and breaking down the essay prompt, it makes sense to spend the remaining 13 minutes reading and annotating the sources.

If you finish reading and annotating early, you can always move on to drafting your synthesis essay. But make sure you're taking your time and reading carefully! It's better to use a little extra time reading and understanding the sources now so that you don't have to go back and re-read the sources later.

body-weightlifting-lift-strong

A strong thesis will do a lot of heavy lifting in your essay. (See what we did there?)

Step 3: Write a Strong Thesis Statement

After you've analyzed the prompt and thoroughly read the sources, the next thing you need to do in order to write a good synthesis essay is write a strong thesis statement .

The great news about writing a thesis statement for this synthesis essay is that you have all the tools you need to do it at your fingertips. All you have to do in order to write your thesis statement is decide what your stance is in relationship to the topic provided.

In the example prompt provided earlier, you're essentially given three choices for how to frame your thesis statement: you can either defend, challenge, or qualify a claim that's been provided by the prompt, that eminent domain is productive and beneficial . Here's what that means for each option:

If you choose to defend the claim, your job will be to prove that the claim is correct . In this case, you'll have to show that eminent domain is a good thing.

If you choose to challenge the claim, you'll argue that the claim is incorrect. In other words, you'll argue that eminent domain isn't productive or beneficial.

If you choose to qualify, that means you'll agree with part of the claim, but disagree with another part of the claim. For instance, you may argue that eminent domain can be a productive tool for governments, but it's not beneficial for property owners. Or maybe you argue that eminent domain is useful in certain circumstances, but not in others.

When you decide whether you want your synthesis essay to defend, challenge, or qualify that claim, you need to convey that stance clearly in your thesis statement. You want to avoid simply restating the claim provided in the prompt, summarizing the issue without making a coherent claim, or writing a thesis that doesn't respond to the prompt.

Here's an example of a thesis statement that received full points on the eminent domain synthesis essay:

Although eminent domain can be misused to benefit private interests at the expense of citizens, it is a vital tool of any government that intends to have any influence on the land it governs beyond that of written law.

This thesis statement received full points because it states a defensible position and establishes a line of reasoning on the issue of eminent domain. It states the author's position (that some parts of eminent domain are good, but others are bad), then goes on to explain why the author thinks that (it's good because it allows the government to do its job, but it's bad because the government can misuse its power.)

Because this example thesis statement states a defensible position and establishes a line of reasoning, it can be elaborated upon in the body of the essay through sub-claims, supporting evidence, and commentary. And a solid argument is key to getting a six on your synthesis essay for AP Lang!

Looking for help studying for your AP exam? Our one-on-one online AP tutoring services can help you prepare for your AP exams. Get matched with a top tutor who got a high score on the exam you're studying for!

Step 4: Create a Bare-Bones Essay Outline

Once you've got your thesis statement drafted, you have the foundation you need to develop a bare bones outline for your synthesis essay. Developing an outline might seem like it's a waste of your precious time, but if you develop your outline well, it will actually save you time when you start writing your essay.

With that in mind, we recommend spending 5 to 10 minutes outlining your synthesis essay . If you use a bare-bones outline like the one below, labeling each piece of content that you need to include in your essay draft, you should be able to develop out the most important pieces of the synthesis before you even draft the actual essay.

To help you see how this can work on test day, we've created a sample outline for you. You can even memorize this outline to help you out on test day! In the outline below, you'll find places to fill in a thesis statement, body paragraph topic sentences, evidence from the sources provided, and commentary :

  • Present the context surrounding the essay topic in a couple of sentences (this is a good place to use what you learned about the major opinions or controversies about the topic from reading your sources).
  • Write a straightforward, clear, and concise thesis statement that presents your stance on the topic
  • Topic sentence presenting first supporting point or claim
  • Evidence #1
  • Commentary on Evidence #1
  • Evidence #2 (if needed)
  • Commentary on Evidence #2 (if needed)
  • Topic sentence presenting second supporting point or claim
  • Topic sentence presenting three supporting point or claim
  • Sums up the main line of reasoning that you developed and defended throughout the essay
  • Reiterates the thesis statement

Taking the time to develop these crucial pieces of the synthesis in a bare-bones outline will give you a map for your final essay. Once you have a map, writing the essay will be much easier.

Step 5: Draft Your Essay Response

The great thing about taking a few minutes to develop an outline is that you can develop it out into your essay draft. After you take about 5 to 10 minutes to outline your synthesis essay, you can use the remaining 30 to 35 minutes to draft your essay and review it.

Since you'll outline your essay before you start drafting, writing the essay should be pretty straightforward. You'll already know how many paragraphs you're going to write, what the topic of each paragraph will be, and what quotations, paraphrases, or summaries you're going to include in each paragraph from the sources provided. You'll just have to fill in one of the most important parts of your synthesis—your commentary.

Commentaries are your explanation of why your evidence supports the argument you've outlined in your thesis. Your commentary is where you actually make your argument, which is why it's such a critical part of your synthesis essay.

When thinking about what to say in your commentary, remember one thing the AP Lang synthesis essay prompt specifies: don't just summarize the sources. Instead, as you provide commentary on the evidence you incorporate, you need to explain how that evidence supports or undermines your thesis statement . You should include commentary that offers a thoughtful or novel perspective on the evidence from your sources to develop your argument.

One very important thing to remember as you draft out your essay is to cite your sources. The AP Lang exam synthesis essay prompt indicates that you can use generic labels for the sources provided (e.g. "Source 1," "Source 2," "Source 3," etc.). The exam prompt will indicate which label corresponds with which source, so you'll need to make sure you pay attention and cite sources accurately. You can cite your sources in the sentence where you introduce a quote, summary, or paraphrase, or you can use a parenthetical citation. Citing your sources affects your score on the synthesis essay, so remembering to do this is important.

body-green-arrow-down

Keep reading for a real-life example of a great AP synthesis essay response!

Real-Life AP Synthesis Essay Example and Analysis

If you're still wondering how to write a synthesis essay, examples of real essays from past AP Lang exams can make things clearer. These real-life student AP synthesis essay responses can be great for helping you understand how to write a synthesis essay that will knock the graders' socks off .

While there are multiple essay examples online, we've chosen one to take a closer look at. We're going to give you a brief analysis of one of these example student synthesis essays from the 2019 AP Lang Exam below!

Example Synthesis Essay AP Lang Response

To get started, let's look at the official prompt for the 2019 synthesis essay:

In response to our society's increasing demand for energy, large-scale wind power has drawn attention from governments and consumers as a potential alternative to traditional materials that fuel our power grids, such as coal, oil, natural gas, water, or even newer sources such as nuclear or solar power. Yet the establishment of large-scale, commercial-grade wind farms is often the subject of controversy for a variety of reasons.

Carefully read the six sources, found on the AP English Language and Composition 2019 Exam (Question 1), including the introductory information for each source. Write an essay that synthesizes material from at least three of the sources and develops your position on the most important factors that an individual or agency should consider when deciding whether to establish a wind farm.

Source A (photo) Source B (Layton) Source C (Seltenrich) Source D (Brown) Source E (Rule) Source F (Molla)

In your response you should do the following:

  • Respond to the prompt with a thesis presents a defensible position.
  • Select and use evidence from at least 3 of the provided sources to support your line of reasoning. Indicate clearly the sources used through direct quotation, paraphrase, or summary. Sources may be cited as Source A, Source B, etc., or by using the description in parentheses.
  • Explain how the evidence supports your line of reasoning.
  • Use appropriate grammar and punctuation in communicating your argument.

Now that you know exactly what the prompt asked students to do on the 2019 AP Lang synthesis essay, here's an AP Lang synthesis essay example, written by a real student on the AP Lang exam in 2019:

[1] The situation has been known for years, and still very little is being done: alternative power is the only way to reliably power the changing world. The draw of power coming from industry and private life is overwhelming current sources of non-renewable power, and with dwindling supplies of fossil fuels, it is merely a matter of time before coal and gas fuel plants are no longer in operation. So one viable alternative is wind power. But as with all things, there are pros and cons. The main factors for power companies to consider when building wind farms are environmental boon, aesthetic, and economic factors.

[2] The environmental benefits of using wind power are well-known and proven. Wind power is, as qualified by Source B, undeniably clean and renewable. From their production requiring very little in the way of dangerous materials to their lack of fuel, besides that which occurs naturally, wind power is by far one of the least environmentally impactful sources of power available. In addition, wind power by way of gearbox and advanced blade materials, has the highest percentage of energy retention. According to Source F, wind power retains 1,164% of the energy put into the system – meaning that it increases the energy converted from fuel (wind) to electricity 10 times! No other method of electricity production is even half that efficient. The efficiency and clean nature of wind power are important to consider, especially because they contribute back to power companies economically.

[3] Economically, wind power is both a boon and a bone to electric companies and other users. For consumers, wind power is very cheap, leading to lower bills than from any other source. Consumers also get an indirect reimbursement by way of taxes (Source D). In one Texan town, McCamey, tax revenue increased 30% from a wind farm being erected in the town. This helps to finance improvements to the town. But, there is no doubt that wind power is also hurting the power companies. Although, as renewable power goes, wind is incredibly cheap, it is still significantly more expensive than fossil fuels. So, while it is helping to cut down on emissions, it costs electric companies more than traditional fossil fuel plants. While the general economic trend is positive, there are some setbacks which must be overcome before wind power can take over as truly more effective than fossil fuels.

[4] Aesthetics may be the greatest setback for power companies. Although there may be significant economic and environmental benefit to wind power, people will always fight to preserve pure, unspoiled land. Unfortunately, not much can be done to improve the visual aesthetics of the turbines. White paint is the most common choice because it "[is] associated with cleanliness." (Source E). But, this can make it stand out like a sore thumb, and make the gargantuan machines seem more out of place. The site can also not be altered because it affects generating capacity. Sound is almost worse of a concern because it interrupts personal productivity by interrupting people's sleep patterns. One thing for power companies to consider is working with turbine manufacturing to make the machines less aesthetically impactful, so as to garner greater public support.

[5] As with most things, wind power has no easy answer. It is the responsibility of the companies building them to weigh the benefits and the consequences. But, by balancing economics, efficiency, and aesthetics, power companies can create a solution which balances human impact with environmental preservation.

And that's an entire AP Lang synthesis essay example, written in response to a real AP Lang exam prompt! It's important to remember AP Lang exam synthesis essay prompts are always similarly structured and worded, and students often respond in around the same number of paragraphs as what you see in the example essay response above.

Next, let's analyze this example essay and talk about what it does effectively, where it could be improved upon, and what score past exam scorers awarded it.

To get started on an analysis of the sample synthesis essay, let's look at the scoring commentary provided by the College Board:

  • For development of thesis, the essay received 1 out of 1 possible points
  • For evidence and commentary, the essay received 4 out of 4 possible points
  • For sophistication of thought, the essay received 0 out of 1 possible points.

This means that the final score for this example essay was a 5 out of 6 possible points . Let's look more closely at the content of the example essay to figure out why it received this score breakdown.

Thesis Development

The thesis statement is one of the three main categories that is taken into consideration when you're awarded points on this portion of the exam. This sample essay received 1 out of 1 total points.

Now, here's why: the thesis statement clearly and concisely conveys a position on the topic presented in the prompt--alternative energy and wind power--and defines the most important factors that power companies should consider when deciding whether to establish a wind farm.

Evidence and Commentary

The second key category taken into consideration when synthesis exams are evaluated is incorporation of evidence and commentary. This sample received 4 out of 4 possible points for this portion of the synthesis essay. At bare minimum, this sample essay meets the requirement mentioned in the prompt that the writer incorporate evidence from at least three of the sources provided.

On top of that, the writer does a good job of connecting the incorporated evidence back to the claim made in the thesis statement through effective commentary. The commentary in this sample essay is effective because it goes beyond just summarizing what the provided sources say. Instead, it explains and analyzes the evidence presented in the selected sources and connects them back to supporting points the writer makes in each body paragraph.

Finally, the writer of the essay also received points for evidence and commentary because the writer developed and supported a consistent line of reasoning throughout the essay . This line of reasoning is summed up in the fourth paragraph in the following sentence: "One thing for power companies to consider is working with turbine manufacturing to make the machines less aesthetically impactful, so as to garner greater public support."

Because the writer did a good job consistently developing their argument and incorporating evidence, they received full marks in this category. So far, so good!

Sophistication of Thought

Now, we know that this essay received a score of 5 out of 6 total points, and the place where the writer lost a point was on the basis of sophistication of thought, for which the writer received 0 out of 1 points. That's because this sample essay makes several generalizations and vague claims where it could have instead made specific claims that support a more balanced argument.

For example, in the following sentence from the 5th paragraph of the sample essay, the writer misses the opportunity to state specific possibilities that power companies should consider for wind energy . Instead, the writer is ambiguous and non-committal, saying, "As with most things, wind power has no easy answer. It is the responsibility of the companies building them to weigh the benefits and consequences."

If the writer of this essay was interested in trying to get that 6th point on the synthesis essay response, they could consider making more specific claims. For instance, they could state the specific benefits and consequences power companies should consider when deciding whether to establish a wind farm. These could include things like environmental impacts, economic impacts, or even population density!

Despite losing one point in the last category, this example synthesis essay is a strong one. It's well-developed, thoughtfully written, and advances an argument on the exam topic using evidence and support throughout.

body-number-four-post-it-note

4 Tips for How to Write a Synthesis Essay

AP Lang is a timed exam, so you have to pick and choose what you want to focus on in the limited time you're given to write the synthesis essay. Keep reading to get our expert advice on what you should focus on during your exam.

Tip 1: Read the Prompt First

It may sound obvious, but when you're pressed for time, it's easy to get flustered. Just remember: when it comes time to write the synthesis essay, read the prompt first !

Why is it so important to read the prompt before you read the sources? Because when you're aware of what kind of question you're trying to answer, you'll be able to read the sources more strategically. The prompt will help give you a sense of what claims, points, facts, or opinions to be looking for as you read the sources.

Reading the sources without having read the prompt first is kind of like trying to drive while wearing a blindfold: you can probably do it, but it's likely not going to end well!

Tip 2: Make Notes While You Read

During the 15-minute reading period at the beginning of the synthesis essay, you'll be reading through the sources as quickly as you can. After all, you're probably anxious to start writing!

While it's definitely important to make good use of your time, it's also important to read closely enough that you understand your sources. Careful reading will allow you to identify parts of the sources that will help you support your thesis statement in your essay, too.

As you read the sources, consider marking helpful passages with a star or check mark in the margins of the exam so you know which parts of the text to quickly re-read as you form your synthesis essay. You might also consider summing up the key points or position of each source in a sentence or a few words when you finish reading each source during the reading period. Doing so will help you know where each source stands on the topic given and help you pick the three (or more!) that will bolster your synthesis argument.

Tip 3: Start With the Thesis Statement

If you don't start your synthesis essay with a strong thesis statement, it's going to be tough to write an effective synthesis essay. As soon as you finish reading and annotating the provided sources, the thing you want to do next is write a strong thesis statement.

According to the CollegeBoard grading guidelines for the AP Lang synthesis essay, a strong thesis statement will respond to the prompt— not restate or rephrase the prompt. A good thesis will take a clear, defensible position on the topic presented in the prompt and the sources.

In other words, to write a solid thesis statement to guide the rest of your synthesis essay, you need to think about your position on the topic at hand and then make a claim about the topic based on your position. This position will either be defending, challenging, or qualifying the claim made in the essay's prompt.

The defensible position that you establish in your thesis statement will guide your argument in the rest of the essay, so it's important to do this first. Once you have a strong thesis statement, you can begin outlining your essay.

Tip 4: Focus on Your Commentary

Writing thoughtful, original commentary that explains your argument and your sources is important. In fact, doing this well will earn you four points (out of a total of six)!

AP Lang provides six to seven sources for you on the exam, and you'll be expected to incorporate quotations, paraphrases, or summaries from at least three of those sources into your synthesis essay and interpret that evidence for the reader.

While incorporating evidence is very important, in order to get the extra point for "sophistication of thought" on the synthesis essay, it's important to spend more time thinking about your commentary on the evidence you choose to incorporate. The commentary is your chance to show original thinking, strong rhetorical skills, and clearly explain how the evidence you've included supports the stance you laid out in your thesis statement.

To earn the 6th possible point on the synthesis essay, make sure your commentary demonstrates a nuanced understanding of the source material, explains this nuanced understanding, and places the evidence incorporated from the sources in conversation with each other. To do this, make sure you're avoiding vague language. Be specific when you can, and always tie your commentary back to your thesis!

body-person-arrows-next

What's Next?

There's a lot more to the AP Language exam than just the synthesis essay. Be sure to check out our expert guide to the entire exam , then learn more about the tricky multiple choice section .

Is the AP Lang exam hard...or is it easy? See how it stacks up to other AP tests on our list of the hardest AP exams .

Did you know there are technically two English AP exams? You can learn more about the second English AP test, the AP Literature exam, in this article . And if you're confused about whether you should take the AP Lang or AP Lit test , we can help you make that decision, too.

Want to improve your SAT score by 160 points or your ACT score by 4 points?   We've written a guide for each test about the top 5 strategies you must be using to have a shot at improving your score. Download them for free now:

Ashley Sufflé Robinson has a Ph.D. in 19th Century English Literature. As a content writer for PrepScholar, Ashley is passionate about giving college-bound students the in-depth information they need to get into the school of their dreams.

Student and Parent Forum

Our new student and parent forum, at ExpertHub.PrepScholar.com , allow you to interact with your peers and the PrepScholar staff. See how other students and parents are navigating high school, college, and the college admissions process. Ask questions; get answers.

Join the Conversation

Ask a Question Below

Have any questions about this article or other topics? Ask below and we'll reply!

Improve With Our Famous Guides

  • For All Students

The 5 Strategies You Must Be Using to Improve 160+ SAT Points

How to Get a Perfect 1600, by a Perfect Scorer

Series: How to Get 800 on Each SAT Section:

Score 800 on SAT Math

Score 800 on SAT Reading

Score 800 on SAT Writing

Series: How to Get to 600 on Each SAT Section:

Score 600 on SAT Math

Score 600 on SAT Reading

Score 600 on SAT Writing

Free Complete Official SAT Practice Tests

What SAT Target Score Should You Be Aiming For?

15 Strategies to Improve Your SAT Essay

The 5 Strategies You Must Be Using to Improve 4+ ACT Points

How to Get a Perfect 36 ACT, by a Perfect Scorer

Series: How to Get 36 on Each ACT Section:

36 on ACT English

36 on ACT Math

36 on ACT Reading

36 on ACT Science

Series: How to Get to 24 on Each ACT Section:

24 on ACT English

24 on ACT Math

24 on ACT Reading

24 on ACT Science

What ACT target score should you be aiming for?

ACT Vocabulary You Must Know

ACT Writing: 15 Tips to Raise Your Essay Score

How to Get Into Harvard and the Ivy League

How to Get a Perfect 4.0 GPA

How to Write an Amazing College Essay

What Exactly Are Colleges Looking For?

Is the ACT easier than the SAT? A Comprehensive Guide

Should you retake your SAT or ACT?

When should you take the SAT or ACT?

Stay Informed

scoring rubric for synthesis essay

Get the latest articles and test prep tips!

Looking for Graduate School Test Prep?

Check out our top-rated graduate blogs here:

GRE Online Prep Blog

GMAT Online Prep Blog

TOEFL Online Prep Blog

Holly R. "I am absolutely overjoyed and cannot thank you enough for helping me!”

pep

Find what you need to study

2024 AP English Language and Composition Exam Guide

12 min read • august 18, 2023

A Q

Attend a live cram event

Review all units live with expert teachers & students

Your guide to the 2024 AP English Language and Composition exam

We know that studying for your AP exams can be stressful, but Fiveable has your back! We created a study plan to help you crush your AP English Language and Composition exam. This guide will continue to update with information about the 2024 exams, as well as helpful resources to help you do your best on test day.  Unlock Cram Mode  for access to our cram events—students who have successfully passed their AP exams will answer your questions and guide your last-minute studying LIVE! And don't miss out on unlimited access to our database of thousands of practice questions. FYI, something cool is coming your way Fall 2023! 👀

Format of the 2024 AP English Language and Composition exam

This year, all AP exams will cover all units and essay types. The 2024 AP English Language and Composition exam format will be:

Section I: Multiple Choice - 45% of your score

45 questions in 1 hour

Section II: Free Response Section - 55% of your score

2 hours and 15 minutes for:

1 synthesis essay

1 rhetorical analysis essay

1 argument essay

Scoring Rubric for the 2024 AP Lang Essays

Synthesis Essay

1 point for a defensible thesis that responds to the prompt

Evidence and Commentary

Max of 4 points for providing evidence from at least 3 sources that support the line of reasoning AND commentary that explains and analyzes the evidence

Sophistication

1 point any of the following:

Creating a nuanced argument

Showing the limitations of the argument

Making effective rhetorical choices

Employing a style that is vivid and persuasive

Rhetorical Analysis Essay

1 point for a defensible thesis that analyzes rhetorical choices

Max of 4 points for providing specific evidence AND consistently explaining how the evidence relates to the line of reasoning AND showing how the rhetorical choices contribute to the author's message .

1 point for any of the following:

Explaining the significance of the rhetorical choices ( rhetorical situation )

Explaining the complexities of the passage and their purpose

Argument Essay

1 point for a defensible thesis

Max of 4 points for providing specific evidence AND consistently explaining the relevance of that evidence .

Crafting a nuanced argument by identifying complexities

Explaining the limitations of the argument by placing it in a broader context

Making rhetorical choices to improve the argument

Check out our study plan below to find resources and tools to prepare for your AP English Language and Composition exam.

When is the 2024 AP English Language and Composition Exam and How Do I Take It?

How should i prepare for the ap lang exam.

First, take stock of your progress in the course so far. What areas have you excelled and which sections need more focus? Download the AP English Language Cheatsheet PDF - a single sheet that covers everything you need to know at a high level. Take note of your strengths and weaknesses!

Build your study plan to review every unit and question type, but focus most on the areas that need the most improvement and practice. We’ve put together this plan to help you study between now and May. This will cover all of the units and essay types to prepare you for your exam

Practice essays are your best friends! The more essays you write, the more automatic the process will come, and the easier the AP exam will be!

Try some of the past exam questions here

We've put together the study plan found below to help you study between now and May. This will cover all of the units and essay types to prepare you for your exam. Pay special attention to the units that you need the most improvement in.

Study, practice, and review for test day with other students during our live cram sessions via  Cram Mode . Cram live streams will teach, review, and practice important topics from AP courses, college admission tests, and college admission topics. These streams are hosted by experienced students who know what you need to succeed.

Pre-Work: Set Up Your Study Environment

Before you begin studying, take some time to get organized.

🖥 Create a study space.

Make sure you have a designated place at home to study. Somewhere you can keep all of your materials, where you can focus on learning, and where you are comfortable. Spend some time prepping the space with everything you need and you can even let others in the family know that this is your study space. 

📚 Organize your study materials.

Get your notebook, textbook, prep books, or whatever other physical materials you have. Also, create a space for you to keep track of review. Start a new section in your notebook to take notes or start a Google Doc to keep track of your notes. Get yourself set up!

📅 Plan designated times for studying.

The hardest part about studying from home is sticking to a routine. Decide on one hour every day that you can dedicate to studying. This can be any time of the day, whatever works best for you. Set a timer on your phone for that time and really try to stick to it. The routine will help you stay on track.

🏆 Decide on an accountability plan.

How will you hold yourself accountable to this study plan? You may or may not have a teacher or rules set up to help you stay on track, so you need to set some for yourself. First, set your goal. This could be studying for x number of hours or getting through a unit. Then, create a reward for yourself. If you reach your goal, then x. This will help stay focused!

2024 AP Lang Study Guide

🚧 unit 1 foundations of rhetoric: analysis of the rhetorical situation and claims ., big takeaways:.

Unit 1 is an introductory unit that lays the foundations for the reading skills associated with how to understand and analyze complex texts. Skills here include identifying the ASPECTS of a text, analyzing the claim given and the evidence used to support that claim, and determining the function of the “chunks” in the argument. Because the content in this unit is very foundational, it is looped throughout the rest of the course instruction.

Definitely do this:

📚 Read these study guides:

Unit 1 Overview: Claims , Reasoning , and Evidence

1.1 Identifying the purpose and intended audience of a text

1.2 Examining how evidence supports a claim

1.3 Developing paragraphs as part of an effective argument

🎥 Watch these videos:

College Board’s Instructional Video: Overview of The Rhetorical Situation .

Fiveable’s How to Read Like an AP Student .

Rhetorical Analysis Thesis Statements  

Rhetorical Analysis Body Paragraphs

✍️ Practice:

Use the Fiveable ASPECTS Guidesheet to help you break down a complex text.

🗺 Can you identify these rhetorical devices?

You won’t be asked to name drop on the exam, but it can be helpful to use devices when discussing strategies. Try this Quizlet to help prepare.

Unit 2 Foundations of Argument: Analysis of an author’s choices in appeals and evidence

Unit 2 is an introductory unit that builds onto the foundations of rhetorical ASPECTS and moves toward planning and writing your own arguments. This unit focuses on the relationships between subject, speaker, and message, including examination of the structure and purpose of the given argument. The unit then moves into the developing thesis statements and building your own arguments with a clear line of reasoning .

Unit 2 Overview: Organizing Information for a Specific Audience

2.1 Analyzing audience and its relationship to the purpose of an argument

2.2 Building an argument with relevant and strategic evidence

2.3 Developing thesis statements

2.4 Developing structure and integrating evidence to reflect a line of reasoning

College Board’s Instructional Video: Identify Rhetorical Situation in a Pre 20th Century Text .

Fiveable’s video on How to Find Rhetorical Devices  

📰 Check out these articles:

Here’s a list of recommended rhetorical devices with definitions and examples!

Use the Fiveable Rhetorical Precis Guidesheet to help you break down a complex text.

🗺 Can you identify these elements of practical argument?

You won’t be asked to name drop of the exam, but it can be helpful to use devices when discussing strategies. Try this Quizlet to help prepare.

👥 Unit 3 Confluence: Synthesis of multiple sources in argumentation

Unit 3 approaches multiple perspectives in argument through the lens of synthesis (that’s FRQ 1). In this study, you learn to identify effective and faulty reasoning while integrating a variety of evidence from credible resources that is properly cited in an original text.

Unit 3 Overview: Perspectives and How Arguments Relate

3.1 Interpreting character description and perspective

3.2 Identifying and avoiding flawed lines of reasoning

3.3 Introducing and integrating sources and evidence

3.4 Using sufficient evidence for an argument

3.5 Attributing and citing references

3.6 Developing parts of a text with cause-effect and narrative methods

Fiveable’s Introduction into Synthesis Essays and How to Begin Your Argument

College Board’s Instructional Video: Complexity in Argument .

🗺 Can you identify these elements of synthesis?

👀 Unit 4 Reasoning : Analysis of argument from introduction to conclusion

Unit 4 includes a greater depth of focus on the writing of effective arguments -- the line of reasoning created in the introduction, built with modes of discourse, and strengthened in the conclusion. An important note about these skills of argumentation is that they build toward all parts of every FRQ. 

Unit 4 Overview: How writers develop arguments, intros, and conclusion

4.1 Developing and connecting thesis statements and lines of reasoning

4.2 Developing introductions and conclusions

4.3 Adjusting an argument to address new evidence

College Board’s Instructional Video: Understanding a Line of Reasoning .

Fiveable’s Effective Annotations .

Try Fiveable’s Guide to LOR Body Paragraphs .

🗺 Can you identify the rhetorical modes?

You won’t be asked to name drop them on the exam, but it can be helpful to use devices when discussing strategies. Try this Quizlet to help prepare.

🧐 Unit 5 Commentary and Analysis: Analysis of complex argument and intentional rhetoric

In Unit 5, the skills look at the minutiae involved in argumentation: development of the line of reasoning that produces strong commentary and maintains the primary claim through all parts of the writing. To achieve these goals, this unit includes a focus on transitions , modifiers , and qualifications for argumentative perspective .  

Unit 5 Overview

5.1 Maintaining ideas throughout an argument

5.2 Developing commentary throughout paragraphs

5.3 Using modifiers to qualify an argument and convey perspective

5.4 Using transitions

Fiveable’s video on How to Improve Analysis Part 1 and Part 2

As well as how to Embed Quotes into Body Paragraphs  

Rhetorical Analysis Body Paragraphs  

Synthesis Essay Body Paragraphs  

Argument Essay Body Paragraphs

Tara Seale’s adaptation for Creating a Line of Reasoning .

🏃‍♂️ Unit 6 Rhetorical Risks: Analysis of multiple perspectives , bias , and shifts with new evidence

In Unit 6, you will notice a direct link building on the ideas of Unit 3 as this instruction looks at position and perspectives while synthesizing information strategically to support a claim.  For greater depth, this unit moves to modify a current argument to include new evidence .

Unit 6 Overview: Position, Perspective , and Bias

6.1 Incorporating multiple perspectives strategically into an argument

6.2 Recognizing and accounting for bias

6.3 Adjusting an argument to new evidence

6.4 Analyzing tone and shifts in tone

College Board’s Instructional Video: Creating a Nuanced Argument .

Fiveable’s video on Tracking an Author’s Argument  

🚀 Unit 7 Complex Argumentation: Analysis of effective arguments, including concession and refutation

The skills of Unit 7 are about putting all units of study together to look at the complexity of a given argument and the effectiveness of the pieces built into that argument.  Though many teachers will have addressed counterarguments, concessions, and refutations before reaching this unit, those skills are highly scrutinized in this segment of learning.

Unit 7 Overview: Successful and Unsuccessful Arguments

7.1 Examining complexities in issues

7.2 Considering how words, phrases, and clauses can modify and limit an argument

7.3 Examining how counterargument or alternative perspectives affect an argument

7.4 Exploring how sentence development affects an argument

Fiveable’s video on Arguments and Counterarguments  

College Board’s Instructional Video: How Argument Demonstrates Understanding .

Check your progress with Fiveable’s AP Language Skills Matrix .

📝 Unit 8 Style: Analysis of how style influences the audience movement

Unit 8 covers how to understand the influence style has on the audience , and the purpose behind each decision. By analyzing these various tactics, students are able to understand the author’s audience , and how to effectively persuade them. Style is an important part in connecting the rest of the course and understanding how the rhetorical choices and devices are used to accomplish a purpose .

Unit 8 Overview: Stylistic Choices

8.1 Choosing comparisons based on an audience

8.2 Considering how sentence development and word choice affect how the writer is perceived by an audience

8.3 Considering how all choices made in an argument affect the audience

8.4 Considering how style affects an argument

Fiveable’s Analysis of the Mindset of the Audience

College Board’s Instructional video: Analyzing and Understanding the Audience

College Board’s explanation of Elements and Context for Style  

Review this quizlet on Elements of Style for more practice.

✏️ Unit 9 Craft: Creation of your own complex argument with synthesis and rhetoric

The final unit of AP Language and Composition covers how to effectively form your own arguments by acknowledging and understanding complexities to create a nuanced and sophisticated argument. It focuses on your ability to comprehend and connect multiple sources to create a well reasoned, and detailed argument as well as how to add in your own rhetorical devices and choices to make your writing more persuasive and effective.

Unit 9 Overview: Developing a Complex Argument

9.1 Strategically conceding, rebutting, or refuting information

9.2 Crafting an argument through stylistic choices like word choice and description

Fiveable’s video on Creating your own Synthesis Arguments

College Board’s video on Complexities within Arguments and How to Create a Nuanced Argument

Key Terms to Review ( 38 )

Argument Structure

Author's Message

Cause-Effect Method

Comparisons

Conclusions

Counterargument

Introductions

Line of Reasoning

Multiple Perspectives

Narrative Method

Objective Reasoning

Perspective

Qualifications

Rhetorical Choices

Rhetorical Situation

Sentence Development

Stylistic Choices

Subjective Reasoning

Textual Evidence

Thesis Development

Thesis Statement

Tone Shifts

Transitions

Word Choice

Fiveable

Stay Connected

© 2024 Fiveable Inc. All rights reserved.

AP® and SAT® are trademarks registered by the College Board, which is not affiliated with, and does not endorse this website.

ESL004: Advanced English as a Second Language

Synthesis essay example and rubric.

In the next section, you will write a synthesis essay in which you will include your ideas on a topic. Here, you will find a sample synthesis essay that will guide you and the rubric that will point out the elements considered in assessing your essay. Carefully examine the information on this page prior to writing your essay. 

This essay example discusses the topic: "Is The Future Paperless?". It synthesizes a variety of viewpoints into a coherent, well-written essay. Notice how the author includes his/her own point of view in paragraph 2? Use this example as a guide to writing a good synthesis essay of your own. Remind yourself that a synthesis is NOT a summary. 

Is going paperless the future? For schools, the answer is likely no, or not for some time. Paper documentation is still critical in the school environment, especially in administration. Student records contain sensitive information, and if online, in a paperless system, these records can be vulnerable to hacking. And while the idea of a school's records being hacked might seem alarmist, recall the recent hack of the United States Office of Personnel Management's hack. Schools might contain similar identifying information and might therefore be tempting to hackers. 

Besides hacking, paper documents continue to have an advantage in established workplaces like schools. There, workflows already incorporate paper documents, and online systems operate only with significant investment in retraining. Students, too, rely on paper. For me, it is easier to get the full picture of an assignment from reading text written on a piece of paper rather than looking at a screen. True that some schools have initiatives in getting iPads and laptops for their students, but these expensive technologies are not as customizable by teachers as paper handouts, so their use is limited. Also, most people would like to have a paper backup in case something happens to their digital device. Paper and document technology are crucial to the current school environment, both in administration and students' own lives. As a company, H.G. Bissinger Office Technology is especially attuned to the significance of paper for education. They recently promoted one of their customer service managers to a new task force on meeting the document technology needs for education. That manager, Lyla Garrity, had created a uniquely strong collaborative relationship with Permian College. Through their work together, she realized that educational document services are an area that specialists could greatly improve, compared to unspecialized, general service that most schools suffer through. H.G. Bissinger Office Technology leases 10 copiers to the Northwest Local School District, along with technical support and copier supplies, excluding paper. For a school, the large investment in a machine is shadowed by the uncertainty of how far from obsolescence a machine might be. Also, purchasing a copier outright will leave the school or business to handle service on its own. Additionally, in these financially limited times, the initial investment of a large sum can be difficult to justify or approve. For schools, uncertainty over future budgets often makes a lease a more flexible option. Most copier leases deal with equipment costs by including provisions in which the client must purchase the machine at the end of the lease. More recently, lease companies like H.G. Bissinger Office Technology are offering leases that are more like rentals. After the monthly fee is paid, the company will take the machine back.

Each of the five items below is worth from 2 to 8 points. To calculate your composite score for your rough draft, add together your scores for all five rubric items below. The maximum score for your final draft is 40 points.

1. Evidential Support

  • Excellent (8 points): I have clearly synthesized the content from the article, paraphrasing the ideas and connecting them to opinions to demonstrate comprehension. All of the main claims in my essay are supported by reasons based on accurate factual evidence derived from the article or a properly-formatted quotation, paraphrase, and/or summary of the assigned text. 
  • Proficient (6 points): I have clearly synthesized the content from the article, paraphrasing the ideas and related topics to demonstrate comprehension; however, my essay does not clearly reflect my opinion on the topic. The majority of the main claims in my essay are backed up by specific factual evidence, although a small number of my claims may be unsubstantiated statements or broad generalizations. When quoting or paraphrasing the assigned reading, I may occasionally misrepresent it or take it out of context. 
  • Adequate (4 points): I have synthesized the content from the article, paraphrasing the ideas and related topics to demonstrate comprehension, but my essay does not mention my point of view on the topic. At least half of the main claims in my essay are based on factual evidence or properly cited passages from the assigned reading. The other half of my claims may be unreasonable, lack quoted or factual support, may be based on misinformation or misreading, may consist of broad generalizations, or may distort and incorrectly format the assigned text.
  • Not Yet Adequate (2 points): I have synthesized some of the content from the article, but my paraphrasing demonstrates limited comprehension of the topic, and my opinion on the topic is not addressed. On balance, most of the claims in my essay are unsubstantiated or based on distortions (or misreadings) of the assigned text. 
  • No Points Awarded (0 points): I have demonstrated minimal synthesis of the topic. My essay does not support its claims with evidence of any kind; my essay does not make claims in response to the prompt.

2. Persuasive Appeals

  • Excellent (8 points): My essay uses a variety of persuasive appeals (emotion, logic, and credibility) to support its claims.
  • Proficient (6 points): My essay uses some of the strategies effectively (as above) some of the time.
  • Adequate (4 points): My essay uses at least one persuasive appeal correctly, but may sometimes use them unfairly or unconvincingly.
  • Not Yet Adequate (2 points): If my essay uses persuasive appeals at all, it does so unfairly or unconvincingly.
  • No Points Awarded (0 points): My essay uses none of the standard persuasive appeals discussed in this course.

3. Rhetorical Strategies

  • Comparison and Contrast
  • Definition of Terms
  • Cause and Effect Analysis
  • Proficient (6 points): My essay uses some of the rhetorical strategies employed by an excellent essay (above); my essay usually uses these strategies with a clear purpose, but may sometimes (for example) define a term without putting it to use, or draw a contrast without showing what it signifies.
  • Adequate (4 points): My essay makes little use of the rhetorical strategies employed by an excellent essay, and may often do so without clear purpose and without using these techniques to persuade my reader; my essay may sometimes use these techniques incorrectly (for example, by providing inaccurate definitions of terms, or by confusing cause and effect).
  • Not Yet Adequate (2 points): My essay incorporates few or no rhetorical appeals, and when it does, it does not use them correctly or persuasively.
  • No Score Awarded (0 points): My essay does not use any of the rhetorical appeals used by an excellent essay (listed above).
  • Excellent (8 points): The grammar errors on the list below, singly or in combination, occur no more than once per 250 words; no persistent patterns of grammar errors are present in the paper; errors do not distract the reader.
  • Proficient (6 points): The errors on the list below, singly or in combination, occur no more than two times per 250 words; single errors from the list below may begin to recur and form a pattern of error; grammar errors are occasionally distracting to the reader.
  • Close to Proficient (4 points): The errors on the list below, singly or in combination, occur on average three times per 250 words; single errors from the list below may recur and form a distinct pattern of error; errors of haste or lack of proofreading are present; grammar errors are persistently distracting to the reader.
  • Not Yet Adequate (2 points): Grammar errors are numerous and impede the reader's comprehension of my essay; my essay reflects a lack of proofreading.

Common Grammatical Errors:

Each error type you have studied is shown next to an example of the error.

  • Inappropriate Punctuation
  • Faulty Parallel Structure
  • Excessive or Inappropriate Use of the Passive Voice
  • Use of weak "to be" verbs rather than strong, active verbs
  • Failure to maintain a formal, rational, objective, unbiased, and academic tone that is directed at an educated audience
  • Proficient (6 points): My essay reads clearly, but may occasionally exhibit one or two of the stylistic errors avoided by an excellent essay (above).
  • Adequate (4 points): Not always, but distractingly often, my essay does not read smoothly because it repeats singly or in combination with the stylistic errors listed above.
  • Not Yet Adequate (2 points): My essay exhibits the stylistic errors above so frequently that it is very difficult to read.

Use this checklist to review each of your sentences for errors:

  • Read each sentence out loud. Do they sound correct? Is anything missing? You can add to your sentences if you want to explain more about your topic.
  • Spelling – Is every word spelled correctly?
  • Correct words – Did you use the right word? Many words in English look similar but have different meanings (for example, like and lick). Check each word to make sure it's the right one.
  • Timeline order – Are your events in the correct order? Make sure your sentences don't jump around.
  • Past tense – Are the verbs in each sentence conjugated in past tense? Go back and review verb endings if you're not sure.
  • Describing words – Do each of your sentences include at least one adjective or one adverb?
  • Capitalization  – The first word in every sentence should be capitalized. After the first word, only proper nouns (like people's names) should be capitalized. Everything else should be lower case.
  • Punctuation  – Does each sentence end with a period? Questions may end with a question mark (?), and exclamations may end with an exclamation mark (!), but most of your sentences should end with a period (.).

Creative Commons License

logo-type-white

AP® English Language

How to score your own ap® english language practice essay.

  • The Albert Team
  • Last Updated On: March 1, 2022

how_to_score_your_own AP® English language practice essay

Practice makes perfect, which makes completing practice free response questions advantageous to the student. Figuring out how you did; however, is more difficult than it seems. As the writer, you have a certain bias that may make it more difficult to grade your own practice essay, but it can be done. If you remain impartial, follow the AP® English Language free response question rubric , and apply the ideas in this guide.

How to Draft a Response

Before we talk about how to score your essay, we must discuss how to draft a response to the AP® English Language free response questions. The first step is to understand your prompt and passage. Next, you must craft a thesis, or your argument. This is vital, because your entire essay should be based around the claim that you present in the thesis. The thesis should contain a roadmap to the rest of your essay, including your supporting details.

Once you have crafted your thesis, then write a short, quick introduction to that thesis, and insert your thesis after the introduction. This introduction must be concise and supplementary to your argument.

In the body paragraphs the thesis is supported. It is recommended that you do this in three body paragraphs at least. Great ways to do this is by citing proof from the passage or passages and inserting your own logical progression. By utilizing the text you allow yourself to gain credibility as a writer and impress your examiners.

Writing drafts

The student will need to complete the three drafts in two hours and fifteen minutes; therefore, it is imperative that the student follows his or her argument and strongly supports it.

If you are practicing writing these free response questions on your own, then it is recommended that you write in a quiet environment that you cannot be disturbed in. This will allow you to focus on the paper as you would in the test location.

Remaining Impartial and Unbiased

When scoring your own AP® English Language free response question essay (FRQ) it is important to be an impartial and unbiased as possible. Be sure to spend at least half an hour away from the essay. This will allow you to clear your mind and be able to see the various mistakes and improvements that can be made to your essay easier.

The best way to do this is by writing the response in the beginning of the week, and then setting it aside until the end of the week. Once you pick the essay back up at the end of the week, then you can read the free response as if you are an outsider scoring your paper. This simulates an examiner reading your paper as it will be done for the AP® English Language scoring.

Be sure to remember that you should not be too easy on yourself. Growth is important with these practice free response questions, and that cannot be done if you deem your paper “perfect”.

Focusing on the AP® English Language Free Response Question Rubric

The next step in scoring your own free response question is to have the AP® English Language Argument Rubric in front of you as you read your essay. By doing this, you will not diverge from the given requirements of the College Board.

Ask yourself questions or make a checklist that contains all of the elements that you will need.

1. Is your grammar and mechanics confusing?

Always be sure to note this, because if your grammar and mechanics are too sloppy or confusing, then your score will fall to a 2. If your use of language is understood but contains major errors, then you will receive a 4 or 5. If your language is tolerable with minimal errors, then you could receive a 6, 7, 8, or 9 depending on the other elements of your essay.

2. How many supporting details do you have? Is your argument supported?

Your argument must be adequately supported. Do you do this in your essay? If there is no evidence of support, then give yourself a 1. Work on bringing in reasoning skills and pulling evidence from your passage.

If your essay reflects few supporting details, then give yourself a 5. This means that you have an argument and supported it, but there is more to be desired. The audience has not bought into your argument yet.

To be able to score yourself with a higher score, your support must be thorough. Citing from the text is extremely important as well as explaining why that quote supports your argument.

3. Is your evidence convincing?

Convincing evidence goes hand in hand with supporting details. Having convincing evidence means that you have utilized your supporting details and explained why they are important. Your purpose is to persuade, and having convincing evidence is vital. The examiner should not doubt the validity of your interpretation, because your evidence must convince the reader.

In order to get an 8 or 9 on the AP® English Language free response questions , you must find textual evidence, use it, and elaborate on its significance to your argument. The last element is especially important as it is the core of your essay.

If you did not relay the significance of your evidence to the argument at all, then give yourself a 4. This means that you have an argument and you have support, but you have not connected the two yet.

If you did relay the significance to the argument somewhere in the essay, then give yourself 5 to a 7 depending on how often you did this.

4. Is your argument clear?

Clarity goes a long way on the AP® English Language free response questions . Your argument must be elevated to the highest priority and explained. This allows the examiner to have no question of what you are claiming.

If you go back and read your essay to find that you are not sure what the argument is, then give yourself a 2. This means that your essay is unsure in your thesis.

To earn a higher score is to be clearer in your argument. Your thesis statement needs to provide a clear claim that you will see and understand every time you read the essay. An essay with a score of eight or nine is direct in its argument and is not subtle in sharing it with the reader. This is the most effective way of delivering the thesis.

5. Do you utilize your sources?

AP® Exam Dates 2018

There is an essay called the synthesis essay which is within the free response question section of the AP® English Language exam. The synthesis essay rubric dictates that you use at least three of the sources in your essay to get a high score.

If you are writing a synthesis essay and you did not include sources, then give yourself a 2. As you utilize sources proficiently your score will rise. It is recommended to use three or more sources; however, be cautious in using more than five. This will seem excessive and your credibility as a proficient analyst will suffer, because the essay will be predominantly the source material and not your own ideas.

6. Are you off topic?

Staying on topic is essential to the free response questions . Never stray from your argument for any reason, because if you are off topic, then your score will drop to a 3 or even may not be scored at all. If you remain on topic, then you have a chance at a much higher score, which will depend on your use of persuasion.

7. Is your writing effectively persuasive overall?

The purpose for writing the essays for the AP® English Language free response questions is to persuade through argumentation and synthesis. Your use of the English language, however, also plays a role in the effectiveness of your response.

Using rhetorical devices and figurative language takes your essay to the next level, and an examiner may bump your score up a number if you are eloquent enough. Therefore, if your essay is especially convincing in its language usage, then take the overall score and raise it one point.

Tips to Remember

There are some elements to keep in mind when you are scoring your own paper. Remember that examiners love to reward students for what they do well. If you see a point that resonated, then keep that in mind as you score yourself.

It is also important to note that the AP® English Language exam’s free response questions are a long and arduous task if you do not practice beforehand. Practice frequently throughout the year to gain the benefits you need and keep on scoring!

Photo by Popular Science Monthly [Public domain], via Wikimedia Commons

By the way, you should check out Albert.io for your AP® English Language review. We have hundreds of AP® English Language practice questions written just for you!

Interested in a school license?​

Popular posts.

AP® Physics I score calculator

AP® Score Calculators

Simulate how different MCQ and FRQ scores translate into AP® scores

scoring rubric for synthesis essay

AP® Review Guides

The ultimate review guides for AP® subjects to help you plan and structure your prep.

scoring rubric for synthesis essay

Core Subject Review Guides

Review the most important topics in Physics and Algebra 1 .

scoring rubric for synthesis essay

SAT® Score Calculator

See how scores on each section impacts your overall SAT® score

scoring rubric for synthesis essay

ACT® Score Calculator

See how scores on each section impacts your overall ACT® score

scoring rubric for synthesis essay

Grammar Review Hub

Comprehensive review of grammar skills

scoring rubric for synthesis essay

AP® Posters

Download updated posters summarizing the main topics and structure for each AP® exam.

Marco Learning

How To Use The New AP® English Scoring Rubric

Last summer, the major changes for the 2019-2020 school year across AP ® courses included a) new unit structures aligned to key course skills, b) teachers gained access to more resources through the launch of AP ® Classroom, and c) many exams underwent major changes. All of these changes were meant to better support AP ® students and teachers in their preparation for – and success on – the May AP ® exams.

Teachers of AP ® English Language and Composition and AP ® English Literature and Composition experienced one of the most significant changes: the introduction of a new analytic rubric for each exam’s Free Response Questions. This new rubric takes the place of the nine-point holistic rubric that has been in use for 20 years.

P.S. Based on feedback from teachers, the College Board made a few final tweaks to the rubrics, which they released on September 30, 2019 . Those changes helped clarify some parts of the rubric. They also released 10 scored student samples for each question of 2018 and 2019 with scoring commentaries!

Using the New AP ® English Rubric With Confidence

Such a significant change can be daunting. If you’ve felt less than prepared to use the new rubrics in your class, the good news is there are major benefits with the new approach.

We’ve been trying it out ourselves – scoring hundreds of previously released student samples alongside a team of expert College Board readers – and we believe the new rubric makes it easier to communicate and teach. That’s exciting news for teachers and students.

The analytic style of this rubric offers clearer direct measures of success. In each scoring category, there are technical requirements to meet, which makes expectations clearer for students and evaluation easier for teachers.

Plus, while development and analysis have always been critical to success on AP ® English work, this rubric offers a more visible focus on evidence and commentary – as well as clarification of exactly what this means.

“There are two aspects of the new rubric I liked most. The first is the analytical nature of the rubric. Since the categories are broken down into three distinct sections, it is easier to consider each portion of this score separately.

Second, i found the new rubric particularly thorough in its explanation. each reporting category clarified what types of responses would or would not earn a specific score for that category. the explanations were more focused and clearer for their descriptions.”, — michael stracco, ap ® english literature reader.

Being big fans of rubrics of all types, but especially analytic rubrics, Marco Learning is here to help. In this post, we break down the big changes and dig into the new rubrics that will be used to evaluate Free Response Questions starting with the May 2020 exams.

Here’s what we will cover in this post:

  • Anatomy of the new rubric
  • How to use the “decision rules” for scoring
  • Implications for teaching
  • How to share the new rubric with your students

Anatomy of the New Rubric

While essays were previously graded on a holistic scale of 0 to 9, reflecting overall quality, the College Board has switched to an analytic rubric, which evaluates student success out of 6 possible points across three scoring categories. The three scoring categories are:

  • Thesis (1 point possible)
  • Evidence and Commentary (4 points possible)
  • Sophistication (1 point possible)

scoring rubric for synthesis essay

Linguistic change! If you’re familiar with the old rubric, you’ll notice a new word choice right away. The new rubric refers to commentary instead of analysis. We perceive this change to be more student friendly, in that it encompasses a broader range of student engagement with the text — rather than simply analyzing the technical aspects of the text/argument, students are encouraged to integrate commentary on important background elements of the text/argument!

Each of these “reporting categories” contains specific requirements that students must meet in order to earn points. The student’s identification and use of evidence continues to be weighted most heavily, with four of the six available points falling within the Evidence and Commentary reporting category.

Here are the core elements of the Synthesis Essay in AP ® English Language:

scoring rubric for synthesis essay

How do the rubrics vary by course and/or essay type?

  • The scoring criteria for Thesis are nearly identical for all six essay types, with a slight modification for AP ® Literature to add that a student’s thesis must “present an interpretation” of the text in question.
  • The Evidence and Commentary scoring criteria have slight variations to address the source of evidence that corresponds to each essay type.
  • The Sophistication scoring criteria are identical across courses and all essay types.

Decision Rules for Scoring

In addition to the basic rubric scoring criteria, the College Board provides helpful “decision rules” for how to apply the criteria more specifically. Notably, these rules vary by essay type.

Access the complete College Board (and revised ) rubrics with the decision rules here:

AP ® English Language

  • Q1: Synthesis Essay
  • Q2: Rhetorical Analysis
  • Q3: Argument Essay

AP ® English Literature

  • Q1: Poetry Analysis
  • Q2: Prose Fiction Analysis
  • Q3: Literary Argument

Applying the Rubric to Student Samples

Many teachers will want to continue to use released exams for student practice (available on the College Board website: AP ® English Language , AP ® English Literature ).

Here’s how to apply the new rubric to your students’ practice timed writings.

Not only is the thesis a vital part of effective written work, it is now a scoring category for AP ® English essays – an explicit requirement. In each essay, students should “respond to the prompt with a thesis that presents a defensible position/interpretation” (the object here varies slightly depending on the question type). Understandably, this must take a position and should go beyond merely restating the prompt or summarizing source texts.

AP ® English Language Argument Essay: Thesis Category

scoring rubric for synthesis essay

Pay special attention to the “additional notes” at the bottom:

First, a thesis located anywhere in the essay may earn the point. While it is typically not good practice for a student to bury their thesis in a conclusion paragraph (because the clarity of their argument may be impacted), a successful concluding thesis would earn the point. When the thesis is not obviously placed in its traditional spot at the end of an introductory paragraph, read closely in case a clear position in response to the prompt is hiding later in the essay.

Second, a thesis may earn a point even if the rest of the response does not support the same line of reasoning . The thesis is evaluated entirely independently from the successful development of the argument.

What changed about THESIS in the revised rubrics?

  • Having a defensible position or interpretation (depending on essay type) matters, but the language around “establishing a line of reasoning” has been removed.
  • Students are not expected to use the thesis to outline their essay. There have been a few scoring notes added, such as that the thesis does not necessarily need to be a single sentence, but the separate sentences need to be in close proximity.

B) EVIDENCE AND COMMENTARY

Worth 4 of the possible 6 points, the Evidence and Commentary category carries the weight of the new rubric. While the source of the evidence varies by essay type, regardless of prompt, students are asked to provide evidence for their position and expand on it with commentary that connects the evidence to their position.

Each rubric’s decision rules include descriptions of “typical” responses that fall into each score level. These descriptions will help you decide how to score a response, but may still prove challenging since you’ll still need to determine how successful a student’s explanation is and where that places it on the rubric.

AP ® English Language Argument Essay: Evidence and Commentary Category

scoring rubric for synthesis essay

As our team of AP ® readers have practiced applying these rules, we have had the most difficulty determining what meets the level of “explanation” in the expectations of the AP ® Language Argument Essay rubric. If a student has provided explanation for their evidence, but not very successfully, for example, they may still be eligible for a score of 3 in this category. This might seem a bit high if you’re oriented to the rigor of the old holistic rubric, but as we’ll explain more below, you’ll need to move away from thinking in terms of the rigor of the old rubric or thinking of essays as “high” or “low.”

If you’re on the fence about a point, we recommend falling back to the classic guidance to reward students for what they do well , particularly in this scoring category. While that specific language has not persisted to this new rubric, based on what we know now, we expect it to persist as a value in College Board scoring on exams.

Note: An essay that does NOT earn the Thesis point is highly unlikely to earn 3 or 4 points in Evidence and Commentary. These higher scores require a clear connection between thesis and evidence.

What changed about EVIDENCE & COMMENTARY in the revised rubrics?

  • Compared to the initial version, the College Board made a helpful structural change: Evidence and Commentary are now discussed independently within the scoring criteria.
  • There is now more focus on supporting all claims for scores of 3 or 4 rather than simply providing examples or evidence that may not be totally successfully linked back to a claim.

C) SOPHISTICATION

We’ve found the Sophistication component requires the most group norming. There are 3-4 “ways” students might demonstrate sophistication of thought listed in the scoring notes, but the scoring criterion is king: the response must, above all, “demonstrate sophistication of thought and/or a complex understanding of the rhetorical situation.”

scoring rubric for synthesis essay

As noted in the rubric, sophistication must be part of the argument , not a passing phrase or reference. While it might be easy to coach students to fulfill one or more of the strategies (“check the box”), it will be very difficult for students to successfully earn the point.

What changed about SOPHISTICATION in the revised rubrics?

  • The College Board has fine-tuned to the decision rules for this point. It is now more clear that this point is very rigorous.
  • There are fewer examples of possible strategies (now 3-4 versus 5-6), notably removing “Utilizing a prose style that is appropriate to the student’s argument”.

scoring rubric for synthesis essay

Teacher tip: The ways a student might demonstrate sophistication may not be obvious for them to include in a response (e.g., using relevant analogies to help an audience better understand an interpretation or discussing alternative interpretations of a text). While we don’t recommend encouraging your students to incorporate the listed strategies to “check the box,” we do recommend encouraging them to be creative in their engagement with the text. And look to these descriptors for teaching ideas!

Avoid These Common Scoring Mistakes

Make sure your scoring is focused on the core areas of the AP ® rubric and doesn’t get caught up by any of these common scoring mistakes.

  • Don’t focus on grammar and mechanics. These aspects of writing are relatively unimportant in scoring. However, if grammatical and/or mechanical errors are so frequent and significant that they interfere with your understanding of the essay, the student is ineligible for the fourth point in the Evidence and Commentary reporting category. It is, however, rare to see this level of technical writing errors in a high-scoring essay.
  • Don’t penalize for “missing” conclusions. While they do make for “nicer” writing, conclusion paragraphs are usually brief and are not actually required in this exam! A student’s conclusion (or the lack of one) is not weighted in the score.
  • Don’t be fooled by flowery writing! Sometimes, a student may write with sophisticated style or flowery language, but fail to adequately analyze evidence or support their argument. While these essays might sound nice, they don’t achieve the main goals of AP ® assignments — namely, the development and analysis of evidence in support of a relevant argument. If an essay is written extremely well on the surface, take a moment to consider whether it meets the assignment goals or if sophisticated styling is masking a lack of analysis. Make sure the specific scoring categories guide your evaluation of each student’s essay!
  • Be careful not to compare the new analytic scoring with the old holistic scores. If you’re very familiar with the 9-point holistic rubric, you may be tempted to continue to consider essays in terms of their success on the old rubric, but the new analytic rubric requires a shift in thinking. Try not to compare the overall scores of students to each other (an overall score of 3 for two students might reflect success in different scoring categories, for example), and be careful about calibrating to past released student samples that were scored on the old scale.

“I believe that the main challenge with the new rubrics is developing a different mindset from the previous holistic scoring. The one for me that seemed most difficult to discard was seeing an essay as either an upper level or a lower level essay. The new rubric discourages such thinking, which many of us experienced scorers often relied upon when first reading through an essay.”

scoring rubric for synthesis essay

Implications for Teaching

We asked Michael Stracco, a long-time English teacher with sixteen years experience as a College Board reader for the AP ® Literature and Composition course, for his advice for teachers when guiding students on the new rubric.

What advice would you give to teachers when guiding students on the new rubric?

Any rubric is going to be a bit formulaic when it comes to preparing students. To the degree that the rubric describes good writing, this new rubric is clearly good teaching of writing. For example, the descriptors of a good thesis sentence are excellent. A teacher would do well to teach a student how to write a good, clear thesis which answers a prompt. However, in years past, it was conceivable that a thesis could be implied on these essays since a stated thesis was not a part of the rubric. Now it is a part of the rubric. Because of this change, all students must now be certain to have a clearly stated thesis. This is a bit formulaic, but it is what teachers must teach in order to prepare their students well for the test.

So this would be my advice to teachers:

  • Remember that you are always a writing teacher. The aspects of writing a literary analysis response are still just teaching how to write well. To be specific, teach students to rely upon the text , to make an analysis of the text, and to elaborate upon that analysis. Be certain to teach students the difference between summary and analysis . This is the heart of scoring well in Row B.
  • I would teach students to put the thesis in the introduction and to underline It always helps for a writer to think about audience in all writing, and in this case the audience is someone who is reading many essays and needs to be certain to see the thesis to assign the point.
  • Regarding a thesis: Teach students that creating a good thesis has two purposes. The first is so that the reader knows where the piece is headed. But the second is so that the writer knows where the piece is headed.
  • Related to this: Planning is essential before writing. You need to know your thesis so that you can keep the piece focused. As an experienced reader, I sometimes would see a student write him/herself into a good essay. The piece would develop into a better essay than the thesis. With a full point being for a good thesis, a student must plan before writing and must write within the context of the thesis or no point would be assigned.

Next Steps for Tackling the New Rubric

As you begin to use the new AP ® English rubrics in your classroom this school year, we encourage you to use the rubric categories and language to guide the skills you teach and follow these next steps:

  • First, download our fillable Teacher English Language Scoring Rubric to keep track of all of these pointers when you are evaluating student work on the new AP ® English rubric. Make copies to reuse with each essay you grade!

TEACHER Scoring Rubric

  • Second, plan a lesson to introduce your students to the new rubric. Review the rubric with them and prep them to use it for self-assessment on their next essay.
  • Third, assign your first practice essay of the school year with confidence! Plan to have your class practice writing thesis statements, and give students plenty of opportunities to practice and workshop more straightforward evidence and commentary strategies.

If your school partners with Marco Learning, take advantage of the opportunity to get personalized feedback for your students from one of our qualified Graders. Graders complete qualification modules to demonstrate proficiency with the new rubric in addition to calibration exercises specific to each prompt.

Log in to your Marco Learning account , and pick from any College Board released prompts dating back to 1999. Scoring and feedback on all AP ® English prompts will be completed using the new rubrics so students and teachers can be prepared for the exams in May!

If we don’t currently work with your school, learn more about how Marco Learning supports AP teachers here .

scoring rubric for synthesis essay

Please read Marco Learning’s Terms and Conditions, click to agree, and submit to continue to your content.

Please read Marco Learning’s Terms and Conditions, click to agree, and submit at the bottom of the window.

MARCO LEARNING TERMS OF USE

Last Modified: 1/24/2023

Acceptance of the Terms of Use

These terms of use are entered into by and between You and Marco Learning LLC (“ Company “, “ we “, or “ us “). The following terms and conditions (these “ Terms of Use “), govern your access to and use of Marco Learning , including any content, functionality, and services offered on or through Marco Learning (the “ Website “), whether as a guest or a registered user.

Please read the Terms of Use carefully before you start to use the Website. By using the Website or by clicking to accept or agree to the Terms of Use when this option is made available to you, you accept and agree to be bound and abide by these Terms of Use. You may not order or obtain products or services from this website if you (i) do not agree to these Terms of Use, or (ii) are prohibited from accessing or using this Website or any of this Website’s contents, goods or services by applicable law . If you do not want to agree to these Terms of Use, you must not access or use the Website.

This Website is offered and available to users who are 13 years of age or older, and reside in the United States or any of its territories or possessions. Any user under the age of 18 must (a) review the Terms of Use with a parent or legal guardian to ensure the parent or legal guardian acknowledges and agrees to these Terms of Use, and (b) not access the Website if his or her parent or legal guardian does not agree to these Terms of Use. By using this Website, you represent and warrant that you meet all of the foregoing eligibility requirements. If you do not meet all of these requirements, you must not access or use the Website.

Changes to the Terms of Use

We may revise and update these Terms of Use from time to time in our sole discretion. All changes are effective immediately when we post them, and apply to all access to and use of the Website thereafter.

These Terms of Use are an integral part of the Website Terms of Use that apply generally to the use of our Website. Your continued use of the Website following the posting of revised Terms of Use means that you accept and agree to the changes. You are expected to check this page each time you access this Website so you are aware of any changes, as they are binding on you.

Accessing the Website and Account Security

We reserve the right to withdraw or amend this Website, and any service or material we provide on the Website, in our sole discretion without notice. We will not be liable if for any reason all or any part of the Website is unavailable at any time or for any period. From time to time, we may restrict access to some parts of the Website, or the entire Website, to users, including registered users.

You are responsible for (i) making all arrangements necessary for you to have access to the Website, and (ii) ensuring that all persons who access the Website through your internet connection are aware of these Terms of Use and comply with them.

To access the Website or some of the resources it offers, you may be asked to provide certain registration details or other information. It is a condition of your use of the Website that all the information you provide on the Website is correct, current, and complete. You agree that all information you provide to register with this Website or otherwise, including but not limited to through the use of any interactive features on the Website, is governed by our Marco Learning Privacy Policy , and you consent to all actions we take with respect to your information consistent with our Privacy Policy.

If you choose, or are provided with, a user name, password, or any other piece of information as part of our security procedures, you must treat such information as confidential, and you must not disclose it to any other person or entity. You also acknowledge that your account is personal to you and agree not to provide any other person with access to this Website or portions of it using your user name, password, or other security information. You agree to notify us immediately of any unauthorized access to or use of your user name or password or any other breach of security. You also agree to ensure that you exit from your account at the end of each session. You should use particular caution when accessing your account from a public or shared computer so that others are not able to view or record your password or other personal information.

We have the right to disable any user name, password, or other identifier, whether chosen by you or provided by us, at any time in our sole discretion for any or no reason, including if, in our opinion, you have violated any provision of these Terms of Use.

Intellectual Property Rights

The Website and its entire contents, features, and functionality (including but not limited to all information, software, text, displays, images, graphics, video, other visuals, and audio, and the design, selection, and arrangement thereof) are owned by the Company, its licensors, or other providers of such material and are protected by United States and international copyright, trademark, patent, trade secret, and other intellectual property or proprietary rights laws. Your use of the Website does not grant to you ownership of any content, software, code, date or materials you may access on the Website.

These Terms of Use permit you to use the Website for your personal, non-commercial use only. You must not reproduce, distribute, modify, create derivative works of, publicly display, publicly perform, republish, download, store, or transmit any of the material on our Website, except as follows:

  • Your computer may temporarily store copies of such materials in RAM incidental to your accessing and viewing those materials.
  • You may store files that are automatically cached by your Web browser for display enhancement purposes.
  • You may print or download one copy of a reasonable number of pages of the Website for your own personal, non-commercial use and not for further reproduction, publication, or distribution.
  • If we provide desktop, mobile, or other applications for download, you may download a single copy to your computer or mobile device solely for your own personal, non-commercial use, provided you agree to be bound by our end user license agreement for such applications.
  • If we provide social media features with certain content, you may take such actions as are enabled by such features.

You must not:

  • Modify copies of any materials from this site.
  • Use any illustrations, photographs, video or audio sequences, or any graphics separately from the accompanying text.
  • Delete or alter any copyright, trademark, or other proprietary rights notices from copies of materials from this site.

You must not access or use for any commercial purposes any part of the Website or any services or materials available through the Website.

If you wish to make any use of material on the Website other than that set out in this section, please contact us

If you print, copy, modify, download, or otherwise use or provide any other person with access to any part of the Website in breach of the Terms of Use, your right to use the Website will stop immediately and you must, at our option, return or destroy any copies of the materials you have made. No right, title, or interest in or to the Website or any content on the Website is transferred to you, and all rights not expressly granted are reserved by the Company. Any use of the Website not expressly permitted by these Terms of Use is a breach of these Terms of Use and may violate copyright, trademark, and other laws.

Trademarks, logos, service marks, trade names, and all related names, logos, product and service names, designs, and slogans are trademarks of the Company or its affiliates or licensors (collectively, the “ Trademarks ”). You must not use such Trademarks without the prior written permission of the Company. All other names, logos, product and service names, designs, and slogans on this Website are the trademarks of their respective owners.

Prohibited Uses

You may use the Website only for lawful purposes and in accordance with these Terms of Use. You agree not to use the Website:

  • In any way that violates any applicable federal, state, local, or international law or regulation (including, without limitation, any laws regarding the export of data or software to and from the US or other countries).
  • For the purpose of exploiting, harming, or attempting to exploit or harm minors in any way by exposing them to inappropriate content, asking for personally identifiable information, or otherwise.
  • To send, knowingly receive, upload, download, use, or re-use any material that does not comply with the Content Standards set out in these Terms of Use.
  • To transmit, or procure the sending of, any advertising or promotional material, including any “junk mail”, “chain letter”, “spam”, or any other similar solicitation.
  • To impersonate or attempt to impersonate the Company, a Company employee, another user, or any other person or entity (including, without limitation, by using email addresses or screen names associated with any of the foregoing).
  • To engage in any other conduct that restricts or inhibits anyone’s use or enjoyment of the Website, or which, as determined by us, may harm the Company or users of the Website or expose them to liability.

Additionally, you agree not to:

  • Use the Website in any manner that could disable, overburden, damage, or impair the site or interfere with any other party’s use of the Website, including their ability to engage in real time activities through the Website.
  • Use any robot, spider, or other automatic device, process, or means to access the Website for any purpose, including monitoring or copying any of the material on the Website.
  • Use any manual process to monitor or copy any of the material on the Website or for any other unauthorized purpose without our prior written consent.
  • Use any device, software, or routine that interferes with the proper working of the Website.
  • Introduce any viruses, Trojan horses, worms, logic bombs, or other material that is malicious or technologically harmful.
  • Attempt to gain unauthorized access to, interfere with, damage, or disrupt any parts of the Website, the server on which the Website is stored, or any server, computer, or database connected to the Website.
  • Attack the Website via a denial-of-service attack or a distributed denial-of-service attack.
  • Otherwise attempt to interfere with the proper working of the Website.

If you use, or assist another person in using the Website in any unauthorized way, you agree that you will pay us an additional $50 per hour for any time we spend to investigate and correct such use, plus any third party costs of investigation we incur (with a minimum $300 charge). You agree that we may charge any credit card number provided for your account for such amounts. You further agree that you will not dispute such a charge and that we retain the right to collect any additional actual costs.

User Contributions

The Website may contain message boards, chat rooms, personal web pages or profiles, forums, bulletin boards, and other interactive features (collectively, “ Interactive Services “) that allow users to post, submit, publish, display, or transmit to other users or other persons (hereinafter, “ post “) content or materials (collectively, “ User Contributions “) on or through the Website.

All User Contributions must comply with the Content Standards set out in these Terms of Use.

Any User Contribution you post to the site will be considered non-confidential and non-proprietary. By providing any User Contribution on the Website, you grant us and our affiliates and service providers, and each of their and our respective licensees, successors, and assigns the right to use, reproduce, modify, perform, display, distribute, and otherwise disclose to third parties any such material for any purpose.

You represent and warrant that:

  • You own or control all rights in and to the User Contributions and have the right to grant the license granted above to us and our affiliates and service providers, and each of their and our respective licensees, successors, and assigns.
  • All of your User Contributions do and will comply with these Terms of Use.

You understand and acknowledge that you are responsible for any User Contributions you submit or contribute, and you, not the Company, have full responsibility for such content, including its legality, reliability, accuracy, and appropriateness.

For any academic source materials such as textbooks and workbooks which you submit to us in connection with our online tutoring services, you represent and warrant that you are entitled to upload such materials under the “fair use” doctrine of copyright law. In addition, if you request that our system display a representation of a page or problem from a textbook or workbook, you represent and warrant that you are in proper legal possession of such textbook or workbook and that your instruction to our system to display a page or problem from your textbook or workbook is made for the sole purpose of facilitating your tutoring session, as “fair use” under copyright law.

You agree that we may record all or any part of any live online classes and tutoring sessions (including voice chat communications) for quality control and other purposes. You agree that we own all transcripts and recordings of such sessions and that these Terms of Use will be deemed an irrevocable assignment of rights in all such transcripts and recordings to us.

We are not responsible or liable to any third party for the content or accuracy of any User Contributions posted by you or any other user of the Website.

Monitoring and Enforcement: Termination

We have the right to:

  • Remove or refuse to post any User Contributions for any or no reason in our sole discretion.
  • Take any action with respect to any User Contribution that we deem necessary or appropriate in our sole discretion, including if we believe that such User Contribution violates the Terms of Use, including the Content Standards, infringes any intellectual property right or other right of any person or entity, threatens the personal safety of users of the Website or the public, or could create liability for the Company.
  • Disclose your identity or other information about you to any third party who claims that material posted by you violates their rights, including their intellectual property rights or their right to privacy.
  • Take appropriate legal action, including without limitation, referral to law enforcement, for any illegal or unauthorized use of the Website.
  • Terminate or suspend your access to all or part of the Website for any or no reason, including without limitation, any violation of these Terms of Use.

Without limiting the foregoing, we have the right to cooperate fully with any law enforcement authorities or court order requesting or directing us to disclose the identity or other information of anyone posting any materials on or through the Website. YOU WAIVE AND HOLD HARMLESS THE COMPANY AND ITS AFFILIATES, LICENSEES, AND SERVICE PROVIDERS FROM ANY CLAIMS RESULTING FROM ANY ACTION TAKEN BY ANY OF THE FOREGOING PARTIES DURING, OR TAKEN AS A CONSEQUENCE OF, INVESTIGATIONS BY EITHER SUCH PARTIES OR LAW ENFORCEMENT AUTHORITIES.

However, we do not undertake to review material before it is posted on the Website, and cannot ensure prompt removal of objectionable material after it has been posted. Accordingly, we assume no liability for any action or inaction regarding transmissions, communications, or content provided by any user or third party. We have no liability or responsibility to anyone for performance or nonperformance of the activities described in this section.

Content Standards

These content standards apply to any and all User Contributions and use of Interactive Services. User Contributions must in their entirety comply with all applicable federal, state, local, and international laws and regulations. Without limiting the foregoing, User Contributions must not:

  • Contain any material that is defamatory, obscene, indecent, abusive, offensive, harassing, violent, hateful, inflammatory, or otherwise objectionable.
  • Promote sexually explicit or pornographic material, violence, or discrimination based on race, sex, religion, nationality, disability, sexual orientation, or age.
  • Infringe any patent, trademark, trade secret, copyright, or other intellectual property or other rights of any other person.
  • Violate the legal rights (including the rights of publicity and privacy) of others or contain any material that could give rise to any civil or criminal liability under applicable laws or regulations or that otherwise may be in conflict with these Terms of Use and our Privacy Policy .
  • Be likely to deceive any person.
  • Promote any illegal activity, or advocate, promote, or assist any unlawful act.
  • Cause annoyance, inconvenience, or needless anxiety or be likely to upset, embarrass, alarm, or annoy any other person.
  • Impersonate any person, or misrepresent your identity or affiliation with any person or organization.
  • Involve commercial activities or sales, such as contests, sweepstakes, and other sales promotions, barter, or advertising.
  • Give the impression that they emanate from or are endorsed by us or any other person or entity, if this is not the case.

(collectively, the “ Content Standards ”)

Copyright Infringement

If you believe that any User Contributions violate your copyright, please contact us  and provide the following information:

  • An electronic or physical signature of the person authorized to act on behalf of the owner of the copyright interest;
  • A description of the copyrighted work that you claim has been infringed;
  • A description of where the material you claim is infringing is located on the website (and such description must reasonably sufficient to enable us to find the alleged infringing material);
  • Your address, telephone number and email address;
  • A written statement by you that you have a good faith belief that the disputed use is not authorized by the copyright owner, its agent, or the law; and
  • A statement by you, made under the penalty of perjury, that the above information in your notice is accurate and that you are the copyright owner or authorized to act on the copyright owner’s behalf.

We may terminate the accounts of any infringers.

Reliance on Information Posted

From time to time, we may make third party opinions, advice, statements, offers, or other third party information or content available on the Website or from tutors under tutoring services (collectively, “Third Party Content”). All Third Party Content is the responsibility of the respective authors thereof and should not necessarily be relied upon. Such third party authors are solely responsible for such content. WE DO NOT (I) GUARANTEE THE ACCURACY, COMPLETENESS OR USEFULNESS OF ANY THIRD PARTY CONTENT ON THE SITE OR ANY VERIFICATION SERVICES DONE ON OUR TUTORS OR INSTRUCTORS, OR (II) ADOPT, ENDORSE OR ACCEPT RESPONSIBILITY FOR THE ACCURACY OR RELIABILITY OF ANY OPINION, ADVICE, OR STATEMENT MADE BY ANY TUTOR OR INSTRUCTOR OR ANY PARTY THAT APPEARS ON THE WEBSITE. UNDER NO CIRCUMSTANCES WILL WE BE RESPONSBILE OR LIABLE FOR ANY LOSS OR DAMAGE RESULTING FROM YOUR RELIANCE ON INFORMATION OR OTHER CONENT POSTED ON OR AVAILBLE FROM THE WEBSITE.

Changes to the Website

We may update the content on this Website from time to time, but its content is not necessarily complete or up-to-date. Any of the material on the Website may be out of date at any given time, and we are under no obligation to update such material.

Information About You and Your Visits to the Website

All information we collect on this Website is subject to our Privacy Policy . By using the Website, you consent to all actions taken by us with respect to your information in compliance with the Privacy Policy.

Online Purchases and Other Terms and Conditions

All purchases through our site or other transactions for the sale of services and information formed through the Website or resulting from visits made by you are governed by our Terms of Sale, which are hereby incorporated into these Terms of Use.

Additional terms and conditions may also apply to specific portions, services, or features of the Website. All such additional terms and conditions are hereby incorporated by this reference into these Terms of Use.

Linking to the Website and Social Media Features

You may link to our homepage, provided you do so in a way that is fair and legal and does not damage our reputation or take advantage of it, but you must not establish a link in such a way as to suggest any form of association, approval, or endorsement on our part without our express written consent.

This Website may provide certain social media features that enable you to:

  • Link from your own or certain third-party websites to certain content on this Website.
  • Send emails or other communications with certain content, or links to certain content, on this Website.
  • Cause limited portions of content on this Website to be displayed or appear to be displayed on your own or certain third-party websites.

You may use these features solely as they are provided by us, and solely with respect to the content they are displayed with and otherwise in accordance with any additional terms and conditions we provide with respect to such features. Subject to the foregoing, you must not:

  • Establish a link from any website that is not owned by you.
  • Cause the Website or portions of it to be displayed on, or appear to be displayed by, any other site, for example, framing, deep linking, or in-line linking.
  • Link to any part of the Website other than the homepage.
  • Otherwise take any action with respect to the materials on this Website that is inconsistent with any other provision of these Terms of Use.

The website from which you are linking, or on which you make certain content accessible, must comply in all respects with the Content Standards set out in these Terms of Use.

You agree to cooperate with us in causing any unauthorized framing or linking immediately to stop. We reserve the right to withdraw linking permission without notice.

We may disable all or any social media features and any links at any time without notice in our discretion.

Links from the Website

If the Website contains links to other sites and resources provided by third parties (“ Linked Sites ”), these links are provided for your convenience only. This includes links contained in advertisements, including banner advertisements and sponsored links. You acknowledge and agree that we have no control over the contents, products, services, advertising or other materials which may be provided by or through those Linked sites or resources, and accept no responsibility for them or for any loss or damage that may arise from your use of them. If you decide to access any of the third-party websites linked to this Website, you do so entirely at your own risk and subject to the terms and conditions of use for such websites.

You agree that if you include a link from any other website to the Website, such link will open in a new browser window and will link to the full version of an HTML formatted page of this Website. You are not permitted to link directly to any image hosted on the Website or our products or services, such as using an “in-line” linking method to cause the image hosted by us to be displayed on another website. You agree not to download or use images hosted on this Website or another website, for any purpose, including, without limitation, posting such images on another website. You agree not to link from any other website to this Website in any manner such that the Website, or any page of the Website, is “framed,” surrounded or obfuscated by any third party content, materials or branding. We reserve all of our rights under the law to insist that any link to the Website be discontinued, and to revoke your right to link to the Website from any other website at any time upon written notice to you.

Geographic Restrictions

The owner of the Website is based in the state of New Jersey in the United States. We provide this Website for use only by persons located in the United States. We make no claims that the Website or any of its content is accessible or appropriate outside of the United States. Access to the Website may not be legal by certain persons or in certain countries. If you access the Website from outside the United States, you do so on your own initiative and are responsible for compliance with local laws.

Disclaimer of Warranties

You understand that we cannot and do not guarantee or warrant that files available for downloading from the internet or the Website will be free of viruses or other destructive code. You are responsible for implementing sufficient procedures and checkpoints to satisfy your particular requirements for anti-virus protection and accuracy of data input and output, and for maintaining a means external to our site for any reconstruction of any lost data. TO THE FULLEST EXTENT PROVIDED BY LAW, WE WILL NOT BE LIABLE FOR ANY LOSS OR DAMAGE CAUSED BY A DISTRIBUTED DENIAL-OF-SERVICE ATTACK, VIRUSES, OR OTHER TECHNOLOGICALLY HARMFUL MATERIAL THAT MAY INFECT YOUR COMPUTER EQUIPMENT, COMPUTER PROGRAMS, DATA, OR OTHER PROPRIETARY MATERIAL DUE TO YOUR USE OF THE WEBSITE OR ANY SERVICES OR ITEMS OBTAINED THROUGH THE WEBSITE OR TO YOUR DOWNLOADING OF ANY MATERIAL POSTED ON IT, OR ON ANY WEBSITE LINKED TO IT.

YOUR USE OF THE WEBSITE, ITS CONTENT, AND ANY SERVICES OR ITEMS OBTAINED THROUGH THE WEBSITE IS AT YOUR OWN RISK. THE WEBSITE, ITS CONTENT, AND ANY SERVICES OR ITEMS OBTAINED THROUGH THE WEBSITE ARE PROVIDED ON AN “AS IS” AND “AS AVAILABLE” BASIS, WITHOUT ANY WARRANTIES OF ANY KIND, EITHER EXPRESS OR IMPLIED. NEITHER THE COMPANY NOR ANY PERSON ASSOCIATED WITH THE COMPANY MAKES ANY WARRANTY OR REPRESENTATION WITH RESPECT TO THE COMPLETENESS, SECURITY, RELIABILITY, QUALITY, ACCURACY, OR AVAILABILITY OF THE WEBSITE. WITHOUT LIMITING THE FOREGOING, NEITHER THE COMPANY NOR ANYONE ASSOCIATED WITH THE COMPANY REPRESENTS OR WARRANTS THAT THE WEBSITE, ITS CONTENT, OR ANY SERVICES OR ITEMS OBTAINED THROUGH THE WEBSITE WILL BE ACCURATE, RELIABLE, ERROR-FREE, OR UNINTERRUPTED, THAT DEFECTS WILL BE CORRECTED, THAT OUR SITE OR THE SERVER THAT MAKES IT AVAILABLE ARE FREE OF VIRUSES OR OTHER HARMFUL COMPONENTS, OR THAT THE WEBSITE OR ANY SERVICES OR ITEMS OBTAINED THROUGH THE WEBSITE WILL OTHERWISE MEET YOUR NEEDS OR EXPECTATIONS.

TO THE FULLEST EXTENT PROVIDED BY LAW, THE COMPANY HEREBY DISCLAIMS ALL WARRANTIES OF ANY KIND, WHETHER EXPRESS OR IMPLIED, STATUTORY, OR OTHERWISE, INCLUDING BUT NOT LIMITED TO ANY WARRANTIES OF MERCHANTABILITY, NON-INFRINGEMENT, AND FITNESS FOR PARTICULAR PURPOSE.

THE FOREGOING DOES NOT AFFECT ANY WARRANTIES THAT CANNOT BE EXCLUDED OR LIMITED UNDER APPLICABLE LAW.

Limitation on Liability

TO THE FULLEST EXTENT PROVIDED BY LAW, IN NO EVENT WILL THE COMPANY, ITS AFFILIATES, OR THEIR LICENSORS, SERVICE PROVIDERS, EMPLOYEES, AGENTS, OFFICERS, OR DIRECTORS BE LIABLE FOR DAMAGES OF ANY KIND, UNDER ANY LEGAL THEORY, ARISING OUT OF OR IN CONNECTION WITH YOUR USE, OR INABILITY TO USE, THE WEBSITE, ANY WEBSITES LINKED TO IT, ANY CONTENT ON THE WEBSITE OR SUCH OTHER WEBSITES, INCLUDING ANY DIRECT, INDIRECT, SPECIAL, INCIDENTAL, CONSEQUENTIAL, OR PUNITIVE DAMAGES, INCLUDING BUT NOT LIMITED TO, PERSONAL INJURY, PAIN AND SUFFERING, EMOTIONAL DISTRESS, LOSS OF REVENUE, LOSS OF PROFITS, LOSS OF BUSINESS OR ANTICIPATED SAVINGS, LOSS OF USE, LOSS OF GOODWILL, LOSS OF DATA, AND WHETHER CAUSED BY TORT (INCLUDING NEGLIGENCE), BREACH OF CONTRACT, OR OTHERWISE, EVEN IF FORESEEABLE.

THE FOREGOING DOES NOT AFFECT ANY LIABILITY THAT CANNOT BE EXCLUDED OR LIMITED UNDER APPLICABLE LAW.

Indemnification

You agree to defend, indemnify, and hold harmless the Company, its affiliates, licensors, and service providers, and its and their respective officers, directors, employees, contractors, agents, licensors, suppliers, successors, and assigns from and against any claims, liabilities, damages, judgments, awards, losses, costs, expenses, or fees (including reasonable attorneys’ fees) arising out of or relating to your violation of these Terms of Use or your use of the Website, including, but not limited to, your User Contributions, any use of the Website’s content, services, and products other than as expressly authorized in these Terms of Use or your use of any information obtained from the Website.

Governing Law and Jurisdiction

All matters relating to the Website and these Terms of Use and any dispute or claim arising therefrom or related thereto (in each case, including non-contractual disputes or claims), shall be governed by and construed in accordance with the internal laws of the State of New Jersey without giving effect to any choice or conflict of law provision or rule (whether of the State of New Jersey or any other jurisdiction).

Any legal suit, action, or proceeding arising out of, or related to, these Terms of Use or the Website shall be instituted exclusively in the federal courts of the United States or the courts of the State of New Jersey in each case located in the County of Monmouth although we retain the right to bring any suit, action, or proceeding against you for breach of these Terms of Use in your country of residence or any other relevant country. You waive any and all objections to the exercise of jurisdiction over you by such courts and to venue in such courts. You may not under any circumstances commence or maintain against us any class action, class arbitration, or other representative action or proceeding.

Arbitration

By using this Website, you agree, at Company’s sole discretion, that it may require you to submit any disputes arising from the use of these Terms of Use or the Website, including disputes arising from or concerning their interpretation, violation, invalidity, non-performance, or termination, to final and binding arbitration under the Rules of Arbitration of the American Arbitration Association applying New Jersey law. In doing so, YOU GIVE UP YOUR RIGHT TO GO TO COURT to assert or defend any claims between you and us. YOU ALSO GIVE UP YOUR RIGHT TO PARTICIPATE IN A CLASS ACTION OR OTHER CLASS PROCEEDING. Your rights may be determined by a NEUTRAL ARBITRATOR, NOT A JUDGE OR JURY. You are entitled to a fair hearing before the arbitrator. The arbitrator can grant any relief that a court can, but you should note that arbitration proceedings are usually simpler and more streamlined than trials and other judicial proceedings. Decisions by the arbitrator are enforceable in court and may be overturned by a court only for very limited reasons.

Any proceeding to enforce this arbitration provision, including any proceeding to confirm, modify, or vacate an arbitration award, may be commenced in any court of competent jurisdiction. In the event that this arbitration provision is for any reason held to be unenforceable, any litigation against Company must be commenced only in the federal or state courts located in Monmouth County, New Jersey. You hereby irrevocably consent to the jurisdiction of those courts for such purposes.

Limitation on Time to File Claims

ANY CAUSE OF ACTION OR CLAIM YOU MAY HAVE ARISING OUT OF OR RELATING TO THESE TERMS OF USE OR THE WEBSITE MUST BE COMMENCED WITHIN ONE (1) YEAR AFTER THE CAUSE OF ACTION ACCRUES, OTHERWISE, SUCH CAUSE OF ACTION OR CLAIM IS PERMANENTLY BARRED.

Waiver and Severability

No waiver by the Company of any term or condition set out in these Terms of Use shall be deemed a further or continuing waiver of such term or condition or a waiver of any other term or condition, and any failure of the Company to assert a right or provision under these Terms of Use shall not constitute a waiver of such right or provision.

If any provision of these Terms of Use is held by a court or other tribunal of competent jurisdiction to be invalid, illegal, or unenforceable for any reason, such provision shall be eliminated or limited to the minimum extent such that the remaining provisions of the Terms of Use will continue in full force and effect.

Entire Agreement

The Terms of Use, our Privacy Policy, and Terms of Sale constitute the sole and entire agreement between you and Marco Learning LLC regarding the Website and supersede all prior and contemporaneous understandings, agreements, representations, and warranties, both written and oral, regarding the Website.

Communications and Miscellaneous

If you provide us your email address, you agree and consent to receive email messages from us. These emails may be transaction or relationship communications relating to the products or services we offer, such as administrative notices and service announcements or changes, or emails containing commercial offers, promotions or special offers from us.

Your Comments and Concerns

This website is operated by Marco Learning LLC, a New Jersey limited liability company with an address of 113 Monmouth Road, Suite 1, Wrightstown, New Jersey 08562.

Please contact us   for all other feedback, comments, requests for technical support, and other communications relating to the Website.

  • help_outline help

iRubric: Synthesis Essay rubric

  • Compare/Contrast Paragraph, rubric, writing components

scoring rubric for synthesis essay

Logo for Open Oregon Educational Resources

APPENDIX A: Sample Grading Rubrics

Discussions.

Total points per discussion: 10

Self-Reflection Journal entries

Total points per self-reflection journal entry: 3

SELF-REFLECTION ESSAY

Draft essays.

This grading rubric is designed for the first draft of an essay. It focuses more on content and organization, and it focuses less on grammar and mechanics.

Total points per draft essay: 10

Revised Essays

This grading rubric is designed for the second draft of an essay. It focuses more on grammar and mechanics, and it focuses less on content and organization.

Total points per revised essay: 10

Synthesis Copyright © 2022 by Timothy Krause is licensed under a Creative Commons Attribution-NonCommercial-ShareAlike 4.0 International License , except where otherwise noted.

IMAGES

  1. 46 Editable Rubric Templates (Word Format) ᐅ TemplateLab

    scoring rubric for synthesis essay

  2. 007 Ap Essay Rubric Example ~ Thatsnotus

    scoring rubric for synthesis essay

  3. How To Use The New AP® English Scoring Rubric

    scoring rubric for synthesis essay

  4. Synthesis Essay Rubric

    scoring rubric for synthesis essay

  5. 46 Editable Rubric Templates (Word Format) ᐅ TemplateLab

    scoring rubric for synthesis essay

  6. Reflection paper rubric

    scoring rubric for synthesis essay

VIDEO

  1. Synthesis essay PROCESS

  2. Writing a Synthesis Essay, Composite Summary and Referencing During Examination/ UGRC210 PastQuo2024

  3. UTILIZATION OF SCORING RUBRIC

  4. AP Physics C: Mechanics FRQ 2017 M1

  5. How to use synthesis repertory !! Best mobile repertory for learners & Students

  6. PRESENTATION PGT202E

COMMENTS

  1. PDF AP English Language and Composition Free-Response Questions Scoring

    AP English Language Scoring Rubric, Free-Response Question 1-3 | SG 1 Scoring Rubric for Question 1: Synthesis Essay 6 points Reporting Category Scoring Criteria Row A Thesis (0-1 points) 4.B 0 points For any of the following: • There is no defensible thesis. • The intended thesis only restates the prompt.

  2. PDF AP Scoring Rubric for Question 1: Synthesis Essay

    Scoring Rubric for Question 1: Synthesis Essay. 0 POINTS. 1 POINT: For any of the following: No defensible thesis Simple restatement of prompt only ... Synthesis Essay. Student's Name: SELF-ASSESSMENT. Effort Assessment. On a scale of 1 to 5, with 1 being low and 5 being high, how .

  3. PDF Scoring Rubric for Question 1: Synthesis Essay (6 points)

    AP English Language and Composition Scoring Rubrics (Effective Fall 2019) September 2019 . Scoring Rubric for Question 1: Synthesis Essay (6 points) Reporting Category Scoring Criteria . Row A Thesis (0-1 points) •

  4. PDF AP English Language and Composition Question 1: Synthesis 2020 Scoring

    Question 1: Synthesis 2020 Scoring Commentaries (Applied to 2018 Student Responses) 2 September 2019 Sample I 6/6 Points (A1 - B4 - C1) Row A: 1/1 The response earned a point for Row A because it presents a thoughtful, multi-sentence thesis that indicates a clear position and establishes a line of reasoning. In paragraph one, the

  5. PDF AP Scoring Rubric for Question 1: Synthesis Essay

    AP® English Language and Composition Scoring Rubric for Question 1: Synthesis Essay Visit www.marcolearning.com for additional learning resources. © Marco Learning ...

  6. PDF AP English Language and Composition Question 1: Synthesis (2019) Sample

    Question 1: Synthesis (2019) Sample Student Responses 1 The student responses in this packet were selected from the 2019 Reading and have been rescored using the new rubrics for 2020. Commentaries for each sample are provided in a separate document. Student responses have been transcribed verbatim; any errors in spelling or grammar appear as ...

  7. PDF AP English Language and Composition FRQ 1 Scoring Commentaries with

    Question 1: Synthesis Scoring Commentaries on 2020 Rubrics (Applied to 2019 Student Responses) 2 September 2019 Sample LL 6/6 Points (A1 - B4 - C1) Row A: 1/1 The response earned a point for Row A because it presents a clear thesis that responds to the prompt and takes a position on the factors that individuals/agencies "ought to ponder."

  8. PDF AP English Language and Composition Free-Response Questions 1-3 Scoring

    Scoring Rubric for Question 1: Synthesis Essay . 6 points. Reporting Category Scoring Criteria. Row A Thesis (0-1 points) 4.B; 0 points: For any of the following: • There is no defensible thesis. • ... AP English Language Scoring Rubric, Free-Response Question 1-3; SG 1:

  9. How to Write the AP Lang Synthesis Essay + Example

    This section counts for 55% of your score and includes the synthesis essay, the rhetorical essay, and the argumentative essay. The synthesis essay requires you to read 6-7 sources and create an argument using at least three sources. The rhetorical analysis essay requires you to describe how a piece of writing evokes specific meanings and symbolism.

  10. AP Lang

    As with all AP exams with free-response questions, the Synthesis question has its own rubric and scoring that we will detail later in this guide. ... A synthesis question is a type of essay prompt that requires students to combine information from multiple sources to develop a cohesive argument or analysis. It tests the student's ability to ...

  11. How to Write a Perfect Synthesis Essay for the AP Language Exam

    Synthesis Essay AP Lang: What It Is and How It Works. The AP Lang synthesis essay is the first of three essays included in the Free Response section of the AP Lang exam. The AP Lang synthesis essay portion of the Free Response section lasts for one hour total. This hour consists of a recommended 15 minute reading period and a 40 minute writing ...

  12. PDF AP Scoring Rubric for Question 1: Synthesis Essay

    Scoring Rubric for Question 1: Synthesis Essay 0 POINTS 1 POINT For any of the following: No defensible thesis Simple restatement of prompt only Summary of topic with no clear claim States an apparent fact rather than a defensible claim. Off-topic Defensible thesis Clear position 0 POINTS 1 POINT 2 POINTS 3 POINTS 4 POINTS Simple restatement of

  13. PDF AP English Language and Composition

    2020 Scoring Guidelines . Question 1: Synthesis Essay 6 points . As the Internet age changes what and how people read, there has been considerable debate about the future of public libraries. While some commentators question whether libraries can stay relevant, others see new possibilities for libraries in the changing dynamics of today's ...

  14. AP Lang Exam Guide

    Section I: Multiple Choice - 45% of your score. 45 questions in 1 hour. Section II: Free Response Section - 55% of your score. 2 hours and 15 minutes for: 1 synthesis essay. 1 rhetorical analysis essay. 1 argument essay. Scoring Rubric for the 2024 AP Lang Essays. Synthesis Essay. Thesis. 1 point for a defensible thesis that responds to the prompt

  15. ESL004: Synthesis Essay Example and Rubric

    In the next section, you will write a synthesis essay in which you will include your ideas on a topic. Here, you will find a sample synthesis essay that will guide you and the rubric that will point out the elements considered in assessing your essay. ... To calculate your composite score for your rough draft, add together your scores for all ...

  16. How to Score Your Own AP® English Language Practice Essay

    There is an essay called the synthesis essay which is within the free response question section of the AP® English Language exam. The synthesis essay rubric dictates that you use at least three of the sources in your essay to get a high score. If you are writing a synthesis essay and you did not include sources, then give yourself a 2. As you ...

  17. How To Use The New AP® English Scoring Rubric

    The Sophistication scoring criteria are identical across courses and all essay types. Decision Rules for Scoring. In addition to the basic rubric scoring criteria, the College Board provides helpful "decision rules" for how to apply the criteria more specifically. Notably, these rules vary by essay type. Access the complete College Board ...

  18. iRubric: Synthesis Essay rubric

    Synthesis essay scoring rubric. Rubric Code: W68BA6. By NancyCook Ready to use Public Rubric Subject: English Type: Writing Grade Levels: 9-12, Undergraduate Desktop Mode Mobile Mode Criteria for Paragraph ...

  19. Synthesis Essay Materials

    The two synthesis essay questions below are examples of the question type that has been one of the three free-response questions on the AP English Language and Composition Exam as of the May 2007 exam. The synthesis question asks students to synthesize information from a variety of sources to inform their own discussion of a topic. Students are given a 15-minute reading period to accommodate ...

  20. APPENDIX A: Sample Grading Rubrics

    DRAFT ESSAYS. This grading rubric is designed for the first draft of an essay. It focuses more on content and organization, and it focuses less on grammar and mechanics. The introduction has an interesting hook, helpful background information, a clear thesis statement, and a preview of the content of the essay.

  21. PDF AP English Language and Composition

    5 Scoring Guidelines 5 Student Samples ... Synthesis Essay 6 points . Vertical farms are indoor agricultural facilities in which plants are grown, often in a hydroponic ( soilless) environment, on tall stacks of shelves. Plants are given water, nutrients, and light mostly through automated processes. Advocates say that vertical farms are key to ...